Sie sind auf Seite 1von 89

500 Question Study by Terry Reynolds, BS, RDCS

1. The parasternal long-axis view allows identification of all the following wall
segments EXCEPT:
A. Basal and mid-interventricular septum
B. Basal and mid-posterior wall of the left ventricle
C. Infero-lateral wall of the left ventricle
D. Left ventricular apex.
2. Wall segments that are visualized in the parasternal short-axis view of the left
ventricle at the level of the papillary muscles include all the following EXCEPT:
A. Anterior septum
B. Anterior wall
C. Antero-inferior wall
D. Antero-lateral wall
3. Left ventricular wall segements that are usually visualized in the apical fourchamber view include all of the following EXCEPT:
A. Anterior wall of the left ventricle
B. Apex
C. Interventricular septumza
D. Lateral wall of left ventricle
4. Wall segments and structures that may be visualized in the apical twochamber view include all of the following EXCEPT:
A. Anterior wall of the left ventricle
B. Coronary sinus
C. Free wall of the right ventricle
D. Inferior wall of the left ventricle
5. The boundaries of the functional left ventricular outflow tract are best
described as extending from the:
A. Anterior aortic valve annulus to the posterior aortic valve annulus
B. Anteromedial position of the tricuspid valve annulus to the pulmonic valve
annulus
C. Free edge of the anterior mitral valve leaflet to the aortic valve annulus
D. Tips of the left ventricular papillary muscles to the edge of the anterior
mitral valve leaflet
6. The landmarks used to identify the anatomic and functional right ventricular
outflow tract are the:
A. Anterior mitral valve leaflet to the left edge of the interventricular septum
B. Aortic valve annulus to the tricuspid valve annulus
C. Tricuspid valve annulus to the aortic valve annulus
D. Tricuspid valve annulus to the pulmonary valve annulus
7. All
A.
B.
C.
D.

of the following are considered atrioventricular valves EXCEPT:


Aortic
Mitral
Bicuspid
Tricuspid

500 Question Study by Terry Reynolds, BS, RDCS


8. All
A.
B.
C.
D.

of the following are names for pulmonic valve leaflets EXCEPT:


Anterior
Left
Non-coronary
Posterior

9. All
A.
B.
C.
D.

the following events are considered to occur in late diastole EXCEPT:


A-dip of the pulmonic valve leaflet on M-mode
A-point of the anterior mitral valve leaflet on M-mode
A-point of the mitral valve Doppler waveform
E-point of the anterior mitral valve leaflet on M-mode

10.The __________ mitral valve leaflet is continuous with the _________ aortic root,
while the _____________ is continuous with the __________ aortic root.
A. Anterior, anterior, posterior mitral valve leaflet, posterior
B. Anterior, posterior, interventricular septum, anterior
C. Posterior, anterior, interventricular septum, posterior
D. Posterior, posterior, anterior mitral valve leafleft, anterior
11.Structures of the mitral valve apparatus include all of the following EXCEPT:
A. Mitral annulus
B. Sinuses of Valsalva
C. Left ventricular walls
D. Papillary muscles
12.The normal mitral valve area is:
A. 1 to 3 cm2
B. 3 to 5 cm2
C. 4 to 6 cm2
D. 7 to 9 cm2

13.All
A.
B.
C.
D.

the following are considered pulmonary vessels EXCEPT:


Main pulmonary artery
Pulmonary capillaries
Pulmonary veins
Vena cava

14.The upper limit of normal for the aortic root diameter in adults as measured
by M-mode echocardiography ranges from:
A. 1.3 to 2.4 cm
B. 2.2 to 2.5 cm
C. 25 to 32 mm
D. 33 to 37 mm

500 Question Study by Terry Reynolds, BS, RDCS


15.The three branches that normally originate from the aortic arch include all the
following EXCEPT:
A. Brachiocephalic artery
B. Left common carotid artery
C. Left subclavian artery
D. Right subclavian artery
16.The point at which the descending aorta and the aortic arch join is called the
aortic:
A. Bifurcation
B. Bulb
C. Isthmus
D. Sinus
17.The thickest layer of an aterial wall is the tunica:
A. Adventitia
B. Intima
C. Media
D. Vasorum
18.The anatomic landmark that demarcates the end section of the descending
thoracic aorta and the beginning of the abdominal aorta is the:
A. Aortic isthmus
B. Diaphragm
C. Ligamentum arteriosum
D. Renal arteries

19.The section of the aorta that is located between the diaphragm and the iliac
arteries is called the:
A. Abdominal aorta
B. Aortic isthmus
C. Descending thoracic aorta
D. Transverse aorta
20.The correct order for the branches of the aortic arch is:
A. Left subclavian, right subclavian, left common carotid
B. Right brachiocephalic, left brachiocephalic, left common carotid
C. Right brachiocephalic, left common carotid, left subclavian
D. Sinus of Valsalva, right innominate, left innominate
21.The normal fetal vascular channel that connects the descending thoracic
aorta and the main pulmonary artery is the:
A. Dustus venosus
B. Ductus arteriosus
C. Ligamentum venosus
D. Ligamentum arteriosum

500 Question Study by Terry Reynolds, BS, RDCS


22.The coronary artery that has a branch called the circumflex artery is the:
A. Left anterior descending coronary artery
B. Left coronary artery
C. Posterior descending coronary artery
D. Right coronary artery
23.The coronary artery that predominantly supplies blood to the right ventricle is
the:
A. Acute marginal branch
B. Anterior descending coronary artery
C. Circumflex coronary artery
D. Left main coronary artery
24.Normal pressure values in millimeters of mercury for the listed cardiac
chambers or great vessels include all the following EXCEPT:
A. Right atrial pressure: 0 to 5 mean
B. Right ventricle: 25 systolic, 0 to 5 diastolic
C. Pulmonary artery: 25 systolic, 10 to 15 diastolic
D. Left ventricle: 120 systolic, 80 diastolic

25.The moderator band is found in the:


A. Right atrium
B. Right ventricle
C. Left atrium
D. Left ventricle
26.The Eustachian valve is found in the:
A. Right atrium
B. Right ventricle
C. Left atrium
D. Left ventricle
27.The Chiari network is found in the:
A. Right atrium
B. Right ventricle
C. Left atrium
D. Left ventricle
28.The normal volume of clear serous fluid in the pericardial sac is:
A. 20 to 50 cc
B. 20 to 50 L
C. 200 to 500 cc
D. 200 to 500 L
29.A potential space behind the left atrium where pericardial effusion could
accumulate is the:
A. Sinus of Valsalva
B. Pleural potential space

500 Question Study by Terry Reynolds, BS, RDCS


C. Oblique sinus
D. Coronary sinus
30.The normal left atrium to right atrium ratio is:
A. 1:1
B. 1.3:1
C. 2:1
D. 3:1
31.The volume or pressure that exists in the ventricle at end-diastole is called:
A. Preload
B. Afterload
C. No-load
D. Sumload
32.The resistance to ejection of blood encountered by the contracting ventricle is
called:
A. Preload
B. Afterload
C. No-load
D. Sumload
33.In general, and outside certain disease states, the greater the stretch of the
muscle cell, the greater the force of contraction.
This principle is called:
A. Franks law of the heart
B. Frank-Starling law of the heart
C. Force-velocity relationship
D. Interval-length relationship
34.The percentage of blood pumped out of the heart per beat is called:
A. Stroke volume
B. Cardiac output
C. Cardiac index
D. Ejection fraction
35.Failed fusion of the superior and inferior endocardial cushions is associated
with all of the following EXCEPT:
A. Partial atrioventricular canal defect
B. Complete atrioventricular canal defect
C. Isolated inlet ventricular septal defect
D. Isolated supracristal ventricular septal defect
36.The imaginary boundaries that define the mid-left ventricle are the:
A. Mitral annulus to the tip of the papillary muscles
B. Base of the papillary muscles to the cardiac apex
C. Tip of the papillary muscles to the base of the papillary muscles
D. Aortic annulus to the edge of the mitral valve

500 Question Study by Terry Reynolds, BS, RDCS


37.The coronary sinus can be differentiated from the descending thoracic aorta
with pulsed-wave Doppler because coronary sinus flow is predominantly
diastolic, while aortic flow is:
A. Equiphasic
B. Phasic
C. Predominantly diastolic
D. Predominantly systolic
38.In standard imaging (M-mode and two-dimensional echocardiography), the
control that determines the amount of electrical energy transmitted to the
transducer from the pulser is:
A. Overall gain
B. Radiofrequency gain
C. Time gain compensation
D. Transmit power
39.Amplification of received signals is controlled by:
A. Output power
B. Overall gain
C. Pulser power
D. Transmit power
40.A method used to compensate for the attenuation of signals as a function of
time/depth along the ultrasound beam is:
A. Overall gain
B. Pulser power
C. Time gain compensation
D. Transmit power
41.The control that selects the minimum signal amplitude to be displayed is:
A. Depth
B. Far gain
C. Output power
D. Reject
42.The control that sets the upper limit to which ultrasound information will be
processed and displayed is:
A. Depth
B. Far gain
C. Overall gain
D. Reject
43.The control used when information from a precise point in the cardiac cycle is
required is the:
A. Electrocardiogram trigger
B. Overall gain
C. Reject
D. Time gain compensation

500 Question Study by Terry Reynolds, BS, RDCS

44.The Doppler control that eliminates low-level frequency shifts is:


A. Filter
B. Gain
C. Output power
D. Sample volume length
45.The control labeled Doppler sample volume depth:
A. Sets the range gate location for pulsed-wave Doppler
B. Does not affect the pulse repetition frequency
C. Does not affect the maximum velocity that can be displayed without
signal aliasing
D. Is useful in continuous-wave Doppler
46.The preferred transducer frequency for imaging a barrel-chested patient is:
A. 2.5 MHz
B. 3.5 MHz
C. 5.0 MHz
D. 7.0 MHz
47.A maneuver that results in a decrease in venous return is:
A. Inspiration
B. Squatting
C. Straight leg raising
D. Valsalva
48.A maneuver that results in an acute increase in blood pressure is:
A. Inhalation of amyl nitrate
B. Supine to standing
C. Isometric handgrip
D. Inspiration
49.A maneuver that will result in tachycardia and a transient decrease in blood
pressure is:
A. Inhalation of amyl nitrate
B. Squatting
C. Standing to supine
D. Straight leg raising
50.A maneuver that will increase venous return is:
A. Expiration
B. Squatting to standing
C. Straight leg raising
D. Supine to standing
51.A maneuver that increases the obstruction in patients with hypertrophic
obstructive cardiomyopathy is:
A. Leg raising
B. Standing to supine
C. Valsalva

500 Question Study by Terry Reynolds, BS, RDCS


D. Inspiration
52.Possible pharmacologic treatments for hypertrophic obstructive
cardiomyopathy include:
A. Propanolol
B. Epinephrine
C. Dobutamine
D. Lasix
53.The method that would best allow visualization of shunt flow across an atrial
septal defect is:
A. M-mode echocardiography
B. Two-dimensional echocardiography
C. Contrast echocardiography
D. Stress echocardiography
54.The most common location for beginning the adult echocardiographic
examination is:
A. Apical
B. Left parasternal
C. Right parasternal
D. Subcostal
55.The three basic planes of a complete two-dimensional echocardiographic
examination are:
A. Long-axis, diagonal-axis, four-chamber
B. Long-axis, short-axis, four-chamber
C. Long-axis, short-axis, subcostal
D. Long-axis, short-axis, two-chamber
56.The two-dimensional echocardiographic view that best visualizes the proximal
segments of the coronary arteries is the:
A. Parasternal long-axis view of the left ventricle
B. Parasternal short-axis view of the aortic valve
C. Parasternal short-axis view of the left ventricle of papillary muscles
D. Subcostal four-chamber view

57.The two-dimensional echocardiographic view that permits examination of the


aortic arch is:
A. Parasternal long axis
B. Apical two chamber
C. Subcostal four chamber
D. Suprasternal long axis
58.The recommended method for determining left ventricular volumes by twodimensional echocardiography is:

500 Question Study by Terry Reynolds, BS, RDCS


A.
B.
C.
D.

Method of discs
Single plane area-length
Bullet method
Prolate ellipse method

59.The ellipsoid single-plane method for determining left ventricular volumes by


two-dimensional echocardiography is also called:
A. Area-length method
B. Bullet method
C. Length-diameter method
D. Simpsons rule
60.A bright horizontal echo that gives the edge of an atrial or ventricular septal
defect a broadened appearance is referred to as the:
A. A sign
B. Fo sign
C. T sign
D. W sign
61.When attempting to determine the presence of a perimembranous ventricular
septal defect in the parasternal long-axis view, the echocardiographer should
tilt the probe:
A. Anteriorly
B. Posteriorly
C. Medially
D. Laterally
62.When attempting to determine whether an inlet ventricular septal defect
from the apical four-chamber view is present, the echocardiographer should
tilt the probe:
A. Anteriorly
B. Posteriorly
C. Medially
D. Laterally
63.The two-dimensional echocardiographic view that is the best approach for
ruling out coarctation of the aorta is:
A. Parasternal long-axis view
B. Parasternal short-axis view of the aortic valve
C. Subcostal five-chamber view
D. Suprasternal long-axis view of the aortic arch
64.The two-dimensional echocardiographic views used to determine whether
truncus arteriosus is present are the parasternal long-axis view and the:
A. Apical five-chamber view
B. Parasternal right ventricular inflow tract view
C. Parasternal short-axis view of the aortic valve
D. Parasternal short-axis view of the left ventricle

500 Question Study by Terry Reynolds, BS, RDCS


65.The two-dimensional view that may be used to directly visualize a patent
ductus arteriosus is the:
A. Parasternal long-axis view of the left ventricle
B. Parasternal short-axis view at the base
C. Apical five-chamber view
D. Subcostal four-chamber view
66.The best two-dimensional view for determining mitral valve area is the:
A. Apical four-chamber view
B. Parasternal long-axis view
C. Parasternal short-axis view
D. Subcostal four-chamber view
67.The gold-standard two-dimensional echocardiographic view for diagnosing
mitral valve prolapsed is the:
A. Apical four-chamber view
B. Parasternal long-axis view
C. Parasternal short-axis view of the mitral valve
D. Subcostal four-chamber
68.Excellent two-dimensional views for imaging the tricuspid valve include all
the following EXCEPT:
A. Parasternal long-axis view
B. Parasternal short-axis view of the aortic valve
C. Apical four-chamber view
D. Subcostal four-chamber view

69.The smallest vegetation that transthoracic two-dimensional echocardiography


can detect is:
A. 1 mm
B. 2 mm
C. 3 mm
D. 4 mm
70.The hump or break occasionally seen on the M-mode of the mitral valve
between the E and F points is designated:
A. B notch
B. f wave
C. Fo
D. h wave
71.The left atrial dimension is measured on M-mode during:
A. Systole
B. End-systole
C. Diastole
D. End-diastole

500 Question Study by Terry Reynolds, BS, RDCS


72.The method used most often for calculating ejection fraction by M-mode
echocardiography is:
A. Ellipsoid biplane method
B. Simpsons method of discs
C. Single-plane area-length method
D. Teichholtz regression
73.A quantitative measure of left ventricular systolic wall thickening is:
A. DT ST DT
B. ST DT DT x 100
C. DT + ST
D. DT + ST DT
74.The top normal left ventricular internal dimension at end diastole in an adult
by M-mode is:
A. 16 mm
B. 26 mm
C. 36 mm
D. 56 mm

75.The normal range for the interventricular septum and the posterior wall of the
left ventricle at end-diastole by M-mode is:
A. 6 to 11 mm
B. 6 to 11 cm
C. 0.6 to .011 mm
D. 60 to 110 mm
76.The E-F slope of the M-mode of the anterior mitral valve leaflet reflects the:
A. Opening rate of the anterior mitral valve leaflet
B. Rate of diastolic filling of the left ventricle
C. Rate of left atrial emptying during diastasis
D. Rate of systolic filling of the left ventricle
77.The E-F slope of the M-mode of the anterior mitral valve leaflet in mitral valve
stenosis is:
A. Decreased
B. Increased
C. Notched
D. Unaffected
78.The mitral valve M-mode points that denote the beginning and the end of
diastole are:
A. C to D
B. D to C
C. D to E
D. E to F

500 Question Study by Terry Reynolds, BS, RDCS


79.The anterior mitral valve leaflet E-F slope is decreased in:
A. Left atrial myxoma
B. Left ventricular volume overload
C. Mitral valve prolapse
D. Mitral valve vegetation
80.The pulmonic valve leadlet most commonly recorded by M-mode is the:
A. Anterior
B. Left
C. Septal
D. Right

81.On M-mode, the abrupt downward motion of the pulmonary valve leaflet
following atrial contraction is called the:
A.
a dip
B. b dip
C. c dip
D. d dip
82.The geometric shape of the right ventricle is:
A. Prolate ellipse
B. Pyramid
C. Rectangle
D. Circle
83.A pressure overload of the right ventricle may produce all the following
echocardiographic findings EXCEPT:
A. D-shaped left ventricle
B. Right ventricular dilatation
C. Right ventricular hypertrophy
D. Small, protected right ventricle
84.A right ventricle volume overload pattern is associated with all the following
echocardiographic findings EXCEPT:
A. Abnormal interventricular septal motion
B. Dilatation of the right ventricle
C. Pancaking of the interventricular septum during ventricular diastole
D. Pancaking of the interventricular septum during ventricular systole
85.The echocardiographic examination that would be the first choice to
delineate a suspected left atrial clot is:
A. Stress echocardiogram
B. Intracardiac echocardiogram
C. Transesophageal echocardiogram
D. Transthoracic echocardiogram

500 Question Study by Terry Reynolds, BS, RDCS


86.The formula used to calculate ejection fraction is:
A. EDD-ESD
B. EDV-ESV
C. EDD-ESD EDD x 100
D. EDV-ESV EVD x 100
87.The formula used for calculating stroke volume is:
A. EDV + ESV
B. EDV - ESV
C. ESV - EDV
D. (EDV ESV) EDV
88.The mean velocity of circumferential fiber shortening (Vcfm) may be
calculated by:
A. EDD ESD EDD
B. EDD ESD LVET x EDD
C. EDV ESV
D. EDV ESV EDV
89.The pressure obtained by a pulmonary artery wedge reflects the pressure in
the:
E. Left atrium
F. Pulmonary artery
G. Right atrium
H. Right ventricle
90.Maneuvers that will increase the duration and severity of mitral valve
prolapsed include all of the following EXCEPT:
A. Inhalation of amyl nitrate
B. Valsalva maneuver
C. Supine to standing
D. Squatting
91.A saline contrast injection to rule out atrial septal defect is performed.
Contrast appears in the left atrium 4 to 8 cycles after the appearance of
contrast in the right atrium. The best explanation is:
A. Left-to-right atrial septal defect shunt
B. Right-to-left atrial septal defect shunt
C. Left ventricle-to-right atrium shunt
D. Pulmonary arteriovenous fistula
92.A maneuver that will increase venous return is:
A. Supine to standing
B. Standing to walking
C. Valsalva maneuver
D. Quiet expiration

500 Question Study by Terry Reynolds, BS, RDCS

93.The effect inspiration has on venous return to the right atrium is:
A. Decrease
B. Increase
C. Depends on the depth of inspiration
D. No effect
94.The recommended maneuver to use when performing a transesophageal
contrast examination in a patient with a possible patent foramen ovale is:
A. Inhalation of amyl nitrate
B. Squatting
C. Supine to standing
D. Valsalva maneuver
95.Possible complications of chronic mitral regurgitation include all the following
EXCEPT:
A. Atrial fibrillation
B. Congestive heart failure
C. Increased risk of sudden death
D. Pulmonary hypertension
96.Congestive heart failure in a patient with significant mitral regurgitation
occurs because of increased pressure in the:
A. Left atrium
B. Right atrium
C. Right ventricle
D. Aorta
97.The left atrial cardiac catheterization pressure tracing in a patient with
significant mitral regurgitation may demonstrate an increase in the:
A. a wave
B. v wave
C. x wave
D. y wave
98.Diastolic mitral regurgitation is associated with:
A. Flail mitral valve
B. Mitral valve prolapse
C. Severe aortic insufficiency
D. Severe tricuspid regurgitation
99.In patients with mitral regurgitation, cardiac catheterization measurements
include all the following EXCEPT:
A. Left ventricular systolic/diastolic pressure
B. Mitral valve area
C. Pulmonary artery pressures
D. Pulmonary capillary wedge pressure

500 Question Study by Terry Reynolds, BS, RDCS


100.

A heart sound associated with significant chronic pure mitral regurgitation is:
A. Loud S1
B. Fixed split S2
C. S3
D. S4

101.A cardiac catherization technique used to determine the severity of mitral


regurgitation is:
A. Cardiac fluoroscopy
B. Coronary arteriography
C. Left ventriculography
D. Supravalvular angiography
102.Possible complications of acute, severe mitral regurgitation include:
A. Syncope
B. Hemoptysis
C. Pulmonary edema
D. Systemic embolization
103.Possible presenting symptoms of significant chronic mitral regurgitation
include:
A. Angina pectoris
B. Ascites
C. Fatigue
D. Syncope
104.A common finding associated with a regurgitant murmur in the elderly is:
A. Aortic valve stenosis
B. Mitral annular calcification
C. Mitral valve stenosis
D. Mitral valve vegetation

105.Chronic mitral regurgitation results in all the following EXCEPT:


A. Left atrial enlargement
B. Left ventricular enlargement
C. Left ventricular volume overload pattern
D. Mitral annular calcification
106.Possible etiologies for mitral regurgitation include all the following EXCEPT:
A. Mitral annulus calcification
B. Mitral valve endocarditis
C. Papillary muscle dysfunction
D. Pulmonary hypertension

500 Question Study by Terry Reynolds, BS, RDCS


107.Possible electrocardiographic findings for patients with significant mitral
regurgitation include all the following EXCEPT:
A. Left atrial enlargement
B. Left ventricular hypertrophy
C. Right atrial enlargement
D. Right ventricular hypertrophy
108.In patients with significant mitral regurgitation, the isovolumic relaxation time
may be:
A. Increased
B. Decreased
C. Affected by respiration
D. Unaffected
109.The effects of chronic mitral regurgitation on interventricular septal motion
include:
A. Akinesis
B. Dyskinesis
C. Hyperkinesis
D. Paradoxical motion
110.M-mode findings associated with significant chronic mitral regurgitation
include all the following EXCEPT:
A. Left atrial enlargement
B. Left ventricular enlargement
C. Fine diastolic flutter of the mitral valve
D. Flying W of the pulmonic valve

111.A two dimensional echocardiographic finding that may indicate significant


chronic mitral regurgitation is:
A. Fine diastolic oscillations of the mitral valve
B. Left ventricular enlargement
C. Left ventricular hypertrophy
D. Premature closure of the mitral valve
112.Systolic bowing of the interatrial septum toward the right atrium may be an
indication of:
A. Atrial septal defect
B. Mitral regurgitation
C. Tricuspid regurgitation
D. Tricuspid stenosis
113.The effect significant mitral regurgitation has on the pulsed-wave Doppler
tracing of the pulmonary veins may be described as:
A. S wave increases, D wave decreases
B. S wave increases, D wave increases
C. S wave decreases, D wave increases

500 Question Study by Terry Reynolds, BS, RDCS


D. Unaffected
114.Possible secondary echocardiographic/Doppler findings in patients with
severe chronic mitral regurgitation include all the following EXCEPT:
A. Increased mitral valve E velocity
B. Increased peak aortic valve velocity
C. Increased right ventricular dimension
D. Shortened time too peak velocity of the right ventricular outflow tract
115.The Doppler finding of mitral valve regurgitation in coronary artery disease is
most likely due to:
A. Flail mitral valve
B. Mitral valve prolapsed
C. Papillary muscle dysfunction
D. Subaortic stenosis
116.An accepted method for the semi-quantitation of mitral regurgitation with
pulsed-wave Doppler is:
A. Mapping technique
B. Maximum velocity of the mitral regurgitation
C. Peak A velocity
D. Pressure half-time
117.An accepted method for determining the severity of mitral regurgitation by
continuous-wave Doppler is spectral:
A. Length
B. Strength
C. Velocity
D. Width
118.A color flow Doppler method for semi-quantitating mitral regurgitation is
regurgitant jet:
A. Area
B. Height
C. Length
D. Turbulence
119.The radius of a mitral regurgitation flow convergence hemisphere is 1.1 cm.
The proximal isovelocity surface area (PISA) is:
A. 1.21 cm2
B. 3.8 cm2
C. 7.6 cm2
D. 15.2 cm2
120.The proximal isovelocity surface area of a mitral regurgitant jet is 7.6 cm 2.
The aliasing flow velocity is 24 cm/sec. The time velocity integral (TVI) of the
mitral regurgitation jet is 150 cm. The maximum velocity of the mitral
regurgitation jet is 580 cm/sec. The mitral regurgitant stroke volume is:
A. 24 cc

500 Question Study by Terry Reynolds, BS, RDCS


B. 47 cc
C. 150 cc
D. 580 cc
121.The peak mitral reguritant velocity reflects the:
A. Direction of the regurgitant jet
B. Etiology of the mitral regurgitation
C. Maximum pressure difference between the left atrium and the left
ventricle
D. Severity of the mitral regurgitation
122.In patients with significant mitral regurgitation, the continuous-wave Doppler
tracing of the regurgitant lesion may demonstrate a(n):
A. Asymmetrical shape of the mitral regurgitation flow velocity spectral
display
B. Jet area of 20%
C. Jet duration of less than 85 msec
D. Symmetrical shape of the mitral regurgitation flow velocity spectral
display
123.Cardiac Doppler evidence of severe mitral regurgitation includes all the
following EXCEPT:
A. Darkly stained continuous-wave Doppler tracing
B. Mitral valve E velocity <1.5 m/sec
C. Pulmonary vein systolic flow reversal
D. Regurgitant jet area >8.0 cm2
124.Flail mitral valve can be differentiated from severe mitral valve prolapsed on
two-dimensional echocardiography because flail mitral valve leaflet
demonstrates:
A. A thicker mitral valve
B. Chronic mitral regurgitation
C. Leaflet tips that point toward the left ventricle
D. Leaflet tips that point toward the left atrium
125.In patients with severe acute mitral regurgitation, the continuous-wave
Doppler maximum
velocity of the regurgitant jet is:
A. Decreased
B. Dependent largely upon left ventricular function
C. Increased
D. Unaffected
126.The most common etiology of mitral valve stenosis is:
A. Congenital
B. Left atrial myxoma
C. Rheumatic fever
D. Severe mitral annular calcification
127.The cardiac valves listed in decreasing order as they are affected by
rheumatic heart disease are:

500 Question Study by Terry Reynolds, BS, RDCS


A. Aortic, pulmonic, tricuspid, and mitral
B. Mitral, aortic, tricuspid, and pulmonic
C. Pulmonic, aortic, tricuspid, and mitral
D. Tricuspid, mitral, pulmonic, and aortic
128. Signs and symptoms of mitral valve stenosis secondary to rheumatic heart
disease include:
A. Angina pectoris
B. Cyanosis
C. Pulmonary Hypertension
D. Vetigo

129. Auscultatory findings for mitral valve stenosis include all the following EXCEPT:
A.
B.
C.
D.

Diastolic rumble at the apex


Loud first heart sound
Opening snap
Systolic ejection murmur heard at the base

130. A 23-year-old woman complaining of dyspnea presents to the cardiologist.


Upon examination, a diastolic rumble and opening snap are heard. The patient
remembers having rheumatic fever at the age of 10. Her electrocardiogram
demonstrated left atrial enlargement and right ventricular hypertrophy. The
diagnosis is:
A.
B.
C.
D.

Aortic valve stenosis


Mitral regurgitation
Rhumatic mitral valve stenosis
Valvular pulmonic valve stenosis

131. Patients with mitral valve stenosis, left atrial enlargement, and atrial fibrillation
are at increased risk for the development of:
A.
B.
C.
D.

Left
Left
Left
Left

atrial myxoma
atrial thrombus
ventricular dilatation
ventricular thrombus

132. The valvular disease with which atrial fibrillation is most commonly associated
is:
A.
B.
C.
D.

Acute aortic insufficiency


Aortic valve stenosis
Mitral valve prolapse
Rheumatic mitral valve stenosis

500 Question Study by Terry Reynolds, BS, RDCS


133. Conditions that may lead to clinical symptoms that mimic those associated
with rheumatic mitral valve stenosis include:
A.
B.
C.
D.

Aortic insufficiency
Left atrial myxoma
Pericardial effusion
Ventricular septal defect

134. Auscultatory findings in rheumatic mitral valve stenosis include:


A.
B.
C.
D.

Austin Flint murmur


Fixed splitting of S2
Loud S1
Mid-systolic click

135. Left atrial thrombus is most often associated with:


A.
B.
C.
D.

Cor triatriatum
Infective endocarditis
Mitral valve prolapsed
Rheumatic mitral valve stenosis

136. In pure rheumatic mitral valve stenosis, the left atrium is _______ and the left
ventricle is _______.
A.
B.
C.
D.

Decreased, decreased
Increased, decreased
Increased, increased
Unchanged, increased

137. The formula used to determine mitral valve area in the cardiac catheterization
laboratory is:
A.
B.
C.
D.

CO
CO
CO
CO

BSA
MPG
DFP 38 x MPG
SEP 44.3 x MPG

138. M-mode findings for the mitral valve in patients with rheumatic mitral valve
stenosis include all the following EXCEPT:
A.
B.
C.
D.

Anterior motion of the posterior mitral valve leaflet


B notch of the anterior mitral valve leaflet
Decreased E-F slope of the anterior mitral valve leaflet
Thickened mitral valve leaflets

500 Question Study by Terry Reynolds, BS, RDCS


139. In the M-mode echocardiogram, features of mitral valve stenosis include all the
following EXCEPT:
A.
B.
C.
D.

Anterior motion of the posterior mitral valve leaflets


Decreased aortic root dimension
Systolic anterior motion of the mitral valve leaflets
Left atrial dilatation

140. In mitral valve stenosis, the posterior mitral valve leaflet on M-mode moves:
A.
B.
C.
D.

Anteriorly
Laterally
Medially
Posteriorly

141. Critical mitral valve stenosis is said to be presented if the mitral valve area is
reduced to:
A.
B.
C.
D.

<1.0 cm2
to 1.5 cm2
1.5 to 2.5 cm2
2.5 to 3.5 cm2

142. Secondary findings in mitral valve stenosis may include:


A.
B.
C.
D.

Left
Left
Left
Left

atrial dilatation and a normal or small left ventricular dimension


atrial dilatation and pulmonary venous stenosis
ventricular and left atrial dilatation
ventricular hypertrophy and left atrial dilatation

143. Typical echocardiographic findings in a patient with isolated rheumatic mitral


valve stenosis include all the following EXCEPT:
A.
B.
C.
D.

D-shaped left ventricle


Dilated left ventricle
Left atrial enlargement
Left atrial thrombus

144. A strong indication for mitral valve stenosis on two-dimensional


echocardiography is an anterior mitral valve leaflet that exhibits:
A.
B.
C.
D.

Coarse, chaotic diastolic motion


Diastolic doming
Reverse doming
Systolic bowing

500 Question Study by Terry Reynolds, BS, RDCS


145. Two-dimensional echocardiographic examination reveals thin mobile mitral
valve leaflet tips and a Doppler E velocity of 1.8 m/sec with a pressure half-time of
180 msec. The most likely diagnosis is:
A.
B.
C.
D.

Abnormal relaxation of the left ventricle


Aortic insufficiency
Moderate to severe mitral annular calcification
Rheumatic mitral stenosis

146. The most accurate method for determining the severity of mitral valve stenosis
is:
A. Determining the maximum velocity across the mitral valve by pulsed-wave
Doppler
B. Measuring the E-F slope of the anterior mitral valve leaflet by M-mode
C. Measuring the thickness of the mitral valve leaflets
D. Performing planimetry of the mitral valve orifice by two-dimensional
echocardiography
147. Two-dimensional echocardiographic findings for rheumatic mitral stenosis
include all the following EXCEPT:
A.
B.
C.
D.

Hockey stick appearance of the anterior mitral valve leaflet


Increased left atrial dimension
Reverse doming of the anterior mitral valve leaflet
Thickened mitral valve leaflets and subvalvular apparatus

148. The classic cardiac Doppler features of mitral valve stenosis include all the
following EXCEPT:
A.
B.
C.
D.

Increased E velocity
Increased mitral valve area
Increased pressure half-time
Turbulent flow

149. The abnormal mitral valve pressure half-time for patients with mitral valve
stenosis is:
A.
B.
C.
D.

0 to 30 msec
30 to 60 msec
60 to 90 msec
90 to 400 msec

150. A deceleration time of 800 msec was obtained by pulsed-wave Doppler in a


patient with rheumatic mitral valve stenosis. The pressure half-time is:
A. 220 msec
B. 232 msec

500 Question Study by Terry Reynolds, BS, RDCS


C. 400 msec
D. 800 msec
151. A Doppler mean pressure gradient across a stenotic mitral valve of 12 mm HG
is obtained. The severity of the mitral stenosis is:
A.
B.
C.
D.

Mild
Moderate
Moderately severe
Severe

152. Mitral stenosis is considered to be severe by all the following criteria EXCEPT:
A.
B.
C.
D.

Mean pressure gradient 12 mmHg


Mitral valve area <1.0 cm2
Mitral valve Doppler A velocity > 1.3 m/sec
Pressure half-time > 220 msec

153. Secondary echocardiographic/Doppler findings in patients with rheumatic


mitral stenosis include all the following EXCEPT:
A.
B.
C.
D.

Abnormal interventricular septal wall motion


Increased right heart dimensions
Increased tricuspid regurgitant jet velocity
Left ventricular dilatation

154. A key word that is often used to describe the characteristics of the valve
leaflets in mitral valve prolapsed is:
A.
B.
C.
D.

Dense
Doming
Redundant
Sclerotic

155. The term myxomatous degeneration is associated with mitral valve:


A.
B.
C.
D.

Flail leaflet
Prolapse
Stenosis
Vegetation

156. The associated auscultatory findings for mitral valve prolapsed include:
A.
B.
C.
D.

Ejection click
Friction rub
Mid-systolic click
Pericardial knock

157. Secondary causes of mitral valve prolapsed include all the following EXCEPT:

500 Question Study by Terry Reynolds, BS, RDCS


A.
B.
C.
D.

Atrial septal defect


Bicuspid aortic valve
Cardiac tamponade
Primary pulmonary hypertension

158. Echocardiographic characteristics of mitral valve prolapsed include all the


following EXCEPT:
A.
B.
C.
D.

Increased mitral valve annulus


Systolic bowing of the mitral valve toward the left atrium
Systolic doming of the leaflets
Thickened, redundant leaflets

159. When performing an echocardiogram on a patient with a thoracic skeletal


abnormality (e.g., pectus excavatum), the echocardiographer must be careful to
rule out:
A.
B.
C.
D.

Atrial septal defect


Bicuspid valve prolapsed
Coarctation of the aorta
Mitral valve prolapsed

160. A redundant mitral valve leaflet is considered present when the leaflet
thickness on M-mode/two-dimensional echocardiography is:
A.
B.
C.
D.

2
3
4
5

mm
mm
mm
mm

161. Mitral valve leaflet chordal rupture usually results in:


A.
B.
C.
D.

Aortic insufficiency
Mitral regurgitation
Pulmonary insufficiency
Tricuspid regurgitation

162. Possible causes of ruptured chordae tendineae of the mitral valve include all
the following EXCEPT:
A.
B.
C.
D.

Carciniod heart disease


Infective endocarditis
Mitral valve prolapsed
Trauma

163. Conditions that may affect the left ventricle in the same way as aortic
insufficiency include all the following EXCEPT:

500 Question Study by Terry Reynolds, BS, RDCS


A.
B.
C.
D.

Atrial septal defect


Mitral regurgitation
Patent ductus arteriosus
Ventricular septal defect

164. Valvular annuloplasty is best accomplished by the following device:


A.
B.
C.
D.

Ballon
Ring
Stent
Coil

165. An M-mode of the aortic root demonstrates fine vibrations in the left atrium.
The most likely cause is:
A.
B.
C.
D.

Flail mitral valve


Left atrial myxoma
Left atrial thrombus
Mitral valve prolapsed

166. On M-mode/two-dimensional echocardiography, dense echoes are noted


posterior to normal mitral valve leaflets. The probable diagnosis is mitral valve:
A.
B.
C.
D.

Annular calcification
Fibrosis
Papilloma
Vegetation

167. The most likely etiology of aortic valve stenosis in a 47-year-old patient is:
A.
B.
C.
D.

Annular
Congenital
Endocarditis
Senile

168. Classic symptoms associated with severe valvular aortic stenosis include all
the following EXCEPT:
A.
B.
C.
D.

Angina pectoris
Atypical chest pain
Congestive heart failure
Syncope

169. An effect of significant aortic valve stenosis on the left ventricle is:
A.
B.
C.
D.

Asymmetrical septal hypertrophy


Concentric left ventricular hypertrophy
Eccentric left ventricular hypertrophy
Protected in significant aortic valve stenosis

500 Question Study by Terry Reynolds, BS, RDCS

170. Pathologies that may result in a left ventricular pressure overload include all
the following EXCEPT:
A.
B.
C.
D.

Discrete subaortic stenosis


Mitral valve stenosis
Systemic hypertension
Valvular aortic stenosis

171. Secondary echocardiographic findings associated with severe valvular aortic


stenosis include all the following EXCEPT:
A.
B.
C.
D.

Decreased left ventricular systolic function (late in course)


Left ventricular hypertrophy
Post-stenotic dilatation of the ascending aorta
Right ventricular hypertrophy

172. The pulse that is characteristic of significant valvular aortic stenosis is:
A.
B.
C.
D.

Pulsus
Pulsus
Pulsus
Pulsus

alternans
bisferiens
paradoxus
parvus et tardus

173. The cardinal symptoms of valvular aortic stenosis include all the following
EXCEPT:
A.
B.
C.
D.

Angina pectoris
Congestive heart failure
Palpitations
Syncope

174. Heart sounds associated with significant valvular aortic stenosis include:
A.
B.
C.
D.

Loud S1
Fixed split S2
S3
S4

175. The murmur of aortic valve stenosis is best described as a:


A.
B.
C.
D.

Holodiastolic decrescendo murmur heard best at the left sterna border


Holosystolic murmur heard at the apex radiating to the axilla
Midsystolic murmur heard best at the right upper sterna border
Midsystolic murmur heard best at the left upper sterna border

500 Question Study by Terry Reynolds, BS, RDCS


176. The principle electrocardiographic finding in severe valvular aortic stenosis is:
A.
B.
C.
D.

Atrial fibrillation
Left atrial enlargement
Left ventricular hypertrophy
Right ventricular hypertrophy

177. The aortic valve area considered critical aortic valve stenosis is:
A.
B.
C.
D.

<
<
<
<

3 cm2
2 cm2
1 cm2
0.75 cm2

178. The formula used to determine aortic valve area in the cardiac catheterization
laboratory is the:
A.
B.
C.
D.

Bernoulli equation
Continuity equation
Doppler equation
Gorlin equation

179. When M-mode evaluation of a systolic ejection murmur reveals thickened


aortic valve leaflets and an aortic cusp excursion of 2.0 cm, the most likely
diagnosis is aortic valve:
A.
B.
C.
D.

Insufficiency
Sclerosis
Stenosis
Vegetation

180. The etiology of aortic valve stenosis includes all the following EXCEPT:
A.
B.
C.
D.

Bacterial
Congenital
Degenerative
Rheumatic

181. The characteristic M-mode findings for aortic valve stenosis include all the
following EXCEPT:
A.
B.
C.
D.

A lack of systolic flutter of the aortic valve leaflets


Diastolic flutter of the aortic valve leaflets
Reduced leaflet separation in systole
Thickening of the aortic valve leaflets

500 Question Study by Terry Reynolds, BS, RDCS


182. The M-mode echocardiogram demonstrates multiple echoes within the aortic
root. The aortic valve excursion as determined by two-dimensional
echocardiography is 7 mm. On physical examination, a crescendo-decrescendo
systolic ejection murmur and a diastolic decrescendo murmur were heard. The most
likely diagnosis is aortic valve:
A.
B.
C.
D.

Insufficiency
Stenosis
Stenosis and aortic insufficiency
Stenosis and mitral valve stenosis

183. Possible two-dimensional echocardiographic findings in significant aortic valve


stenosis include all the following EXCEPT:
A.
B.
C.
D.

Aortic valve calcification


Left ventricular hypertrophy
Post-stenosic dilatation of the ascending aorta
Post-stenotic dilatation of the descending aorta

184. In the parasternal long-axis view, severe aortic valve stenosis is defined as an
aortic valve leaflet separation that measures:
A.
B.
C.
D.

14 mm
12 mm
10 mm
8 mm

185. Cardiac Doppler parameters used to assess the severity of valvular aortic
stenosis include all the following EXCEPT:
A.
B.
C.
D.

Aortic pressure half-time


Aortic velocity ratio
Mean pressure gradient
Peak aortic valve velocity

186. The intracardiac pressure that will most likely be increased in patients with
severe valvular aortic stenosis is:
A.
B.
C.
D.

Left ventricular pressure at end-diastole


Pulmonary artery pressure
Right atrial pressure
Right ventricular pressure at end-diastole

187. The onset of flow to peak aortic velocity Doppler tracing in severe valvular
aortic stenosis is:

500 Question Study by Terry Reynolds, BS, RDCS


A.
B.
C.
D.

Increased
Decreased
Decreased with expiration
Increased with inspiration

188. A Doppler mean pressure gradient of 18 mmHg is calculated in a patient with


valvular aortic stenosis. The severity of the stenosis is:
A.
B.
C.
D.

Mild
Moderate
Moderately severe
Severe

189. The severity of aortic valve stenosis may be underestimated if only the
maximum velocity measurement is used in the following condition:
A.
B.
C.
D.

Anemia
Doppler intercept angle of 0
Low cardiac output
Significant aortic insufficiency

190. The two-dimensional view that best visualizes systolic doming of the aortic
valve leaflets is the:
A.
B.
C.
D.

Apical five-chamber view


Parasternal long-axis view
Parasternal short-axis view of the aortic valve
Subcostal short-axis view of the aortic valve

191. Of the three pressure gradients that can be measured in the cardiac
catheterization laboratory, the largest is:
A.
B.
C.
D.

Maximum peak instantaneous gradient


Mean transvalvular systolic gradient
Peak-to-peak gradient
Peak-to-mean gradient

192. Of the transvalvular pressure gradients that can be measured in the


echocardiography laboratory, the most useful in examining aortic valve stenosis is
probably:
A.
B.
C.
D.

Mean diastolic gradient


Mean systolic gradient
Peak instantaneous pressure gradient
Peak-to-peak gradient

193. The echocardiographer may differentiate between the similar systolic flow
pattern seen in coexisting severe aortic valve stenosis and mitral regurgitation by
all the following methods EXCEPT:

500 Question Study by Terry Reynolds, BS, RDCS


A. The aortic ejection time is shorter than the mitral regurgitation time
B. Mitral regurgitation flow always lasts until mitral valve opening, whereas
aortic valve stenosis flow does not
C. Mitral diastolic filling profile should be present during recording of the mitral
regurgitation, whereas no diastolic flow is observed in aortic valve stenosis
D. Since both are systolic flow patterns, it is not possible to separate mitral
regurgitation from aortic valve stenosis
194. The most common etiology for chronic aortic insufficiency is:
A.
B.
C.
D.

Idiopathic dilatation of the aortic root and aortic annulus


Infective endocarditis
Marfans syndrome
Trauma

195. The characteristic feature of the murmur of aortic insufficiency is a:


A. Diastolic decrescendo blowing murmur heard best along the left sterna
border
B. Diastolic crescendo-decrescendo murmur heard best along the left upper
sterna border
C. Diastolic rumble following an opening snap
D. Harsh systolic ejection murmur heard best at the right upper sterna border
196. The murmur associated with severe aortic insufficiency is:
A.
B.
C.
D.

Austin Flint murmur


Carvallos murmur
Graham Steell murmur
Stills murmur

197. A technique used in the cardiac catheterization laboratory that determines the
severity of aortic insufficiency is:
A.
B.
C.
D.

Austin Flint technique


Judkins technique
Left ventriculography
Supravalvular aortography

198. In significant chronic aortic insufficiency, M-mode/two-dimensional evidence


includes all the following EXCEPT:
A. Hyperkinesis of the interventricular septum
B. Hyperkinesis of the posterior wall of the ledt ventricle

500 Question Study by Terry Reynolds, BS, RDCS


C. Left ventricular dilatation
D. Paradoxical interventricular septal motion
199. Low-frequency diastolic fluttering of the aortic valve closure line on the Mmode of the aortic valve is:
A.
B.
C.
D.

A normal finding
Indicative of cusp rupture or flail aortic valve
Pathognomonic for bicuspid aortic valve
Pathognomonic for significant aortic valve stenosis

200. Septal motion in significant aortic insufficieny often demonstrates:


A.
B.
C.
D.

Anterior motion during systole


Exaggerated early diastolic dip
Normal motion
Thinning with dyskinesis during systole

201. Premature closure of the mitral valve is associated with all the following
EXCEPT:
A.
B.
C.
D.

Acute severe mitral regurgitation


Acute severe aortic insufficiency
First-degree atrioventricular block
Loss of sinus rhythm

202. Echocardiographic evidence of severe, acute aortic insufficiency includes all


the following EXCEPT:
A.
B.
C.
D.

Premature closure of the mitral valve


Premature opening of the aortic valve
Premature opening of the mitral valve
Reverse doming of the anterior mitral valve leaflet

203. The M-mode measurements that have been proposed as an indicator for aortic
valve replacement in patients with chronic severe aortic insufficiency are left
ventricular:
A.
B.
C.
D.

End-diastolic dimension 55 mm and fractional shortening of 25%


End-diastolic dimension 55 mm and fractional shortening of 25%
End-diastolic dimension 70 mm and left atrial dimension 55 mm
End-systolic dimension 55 mm and fractional shortening of 25%

204. Reverse diastolic doming of the anterior mitral valve leaflet is associated with:
A. Flail mitral valve
B. Papillary muscle dysfunction
C. Rheumatic mitral valve stenosis

500 Question Study by Terry Reynolds, BS, RDCS


D. Severe aortic insufficiency
205. The hallmark M-mode finding for aortic insufficiency is:
A.
B.
C.
D.

Coarse diastolic flutter of the anterior mitral valve leaflet


Fine diastolic flutter of the anterior mitral valve leaflet
Chaotic diastolic flutter of the mitral valve
Systolic flutter of the aortic valve

206. A pulsed-wave Doppler blood flow velocity profile of aortic insufficiency


obtained from the apical five-chamber view will demonstrate a diastolic:
A.
B.
C.
D.

Laminar high-velocity flow signal


Laminar low-velocity flow signal
Turbulent high-velocity flow signal
Turbulent low-velocity flow signal

207. The simplest semiquantitative technique for determining the severity of aortic
insufficiency using pulsed-wave Doppler is:
A. Comparing the detected jet height to the left ventricular outflow tract height
B. Detecting a laminar diastolic flow pattern, which indicates severe aortic
insufficiency
C. Examining the spectral strength of the regurgitant jet
D. Flow mapping of the left ventricle
208. Severe aortic insufficiency can be diagnosed by continuous-wave Doppler by
all the following criteria EXCEPT:
A.
B.
C.
D.

A maximum velocity of > 3 m/sec


A pressure half-time of 300 msec
Aortic insufficiency deceleration slope 3 m/sec
Darkened spectrum of the regurgitant jet

209. The severity of aortic insufficiency may best be determined with color flow
Doppler by the following method:
A. Measuring the aortic insufficiency jet aliasing area in the parasternal longaxis view
B. Comparing the aortic insufficiency jet height with the left ventricular outflow
tract height
C. Measuring the aortic insufficiency jet maximal height
D. Noting the temporal pattern of color variance
210. A color flow Doppler technique that permits detection of 3+ to 4+ aortic
insufficiency is:
A. Early diastolic flow reversal in the abdominal aorta
B. Early diastolic flow reversal in the descending thoracic aorta
C. Holodiastolic flow reversal in the descending thoracic aorta

500 Question Study by Terry Reynolds, BS, RDCS


D. Holosystolic flow reversal in the abdominal aorta
211. Proximal flow convergence of an aortic insufficiency jet as seen on color flow
Doppler may represent:
A.
B.
C.
D.

Physiologic insufficiency
Mild (1+) aortic insufficiency
Moderate (2+) aortic insufficiency
Moderately severe (3+ 4+) aortic insufficiency

212. The mitral valve inflow pattern often associated with severe acute aortic
insufficiency is stage:
A.
B.
C.
D.

I
II
III
IV

213. The Doppler signal of aortic insufficiency may be differentiated from the
Doppler signal of mitral stenosis by the following guideline:
A. If the diastolic flow pattern commences before mitral valve opening, then the
signal is due to aortic insufficiency
B. If the diastolic flow pattern commences after mitral valve opening, then the
signal is due to aortic insufficiency
C. The Doppler flow velocity pattern of mitral valve stenosis is laminar, while the
Doppler flow velocity pattern of aortic insufficiency is turbulent
D. Since both mitral valve stenosis and aortic insufficiency are diastolic, it is not
possible to differentiate the Doppler flow velocity patterns.
214. The M-mode finding that indicates severe acute aortic insufficiency is
premature aortic valve:
A.
B.
C.
D.

Closure
Diastolic flutter
Mid-systolic closure
Opening

215. Posterior displacement of aortic valve leaflet(s) into the left ventricular outflow
tract during diastole is called aortic valve:
A.
B.
C.
D.

Prolapsed
Sclerosis
Stenosis
Vegetation

500 Question Study by Terry Reynolds, BS, RDCS


216. A two-dimensional echocardiographic finding in a patient with pure aortic
insufficiency is:
A.
B.
C.
D.

Left atrial enlargement


Left ventricular enlargement
Right atrial enlargement
Right ventricular hypertrophy

217. High-frequency diastolic flutter of the aortic valve with echoes extending into
the left ventricular outflow tract during diastole on M-mode echocardiographic
represents:
A.
B.
C.
D.

Ascending aortic aneurysm


Flail aortic valve leaflet
Valvular aortic sclerosis
Valvular aortic stenosis

218. The most common etiology of acute aortic insufficiency is:


A.
B.
C.
D.

Aortic ballon valvuloplasty


Hypertension
Infective endocarditis
Rheumatic fever

219. A pulse that is associated with significant aortic insufficiency is:


A.
B.
C.
D.

Pulsus
Pulsus
Pulsus
Pulsus

alternas
bisferiens
paradoxus
parvus et tardus

220. M-mode reveals diastolic flutter of the anterior mitral valve and a left
ventricular end-systolic dimension of 58 mm. Two-dimensional echocardiography
demonstrates an aortic root that is 4.5 cm in diameter with aortic valve sclerosis.
The aortic insufficiency jet is mapped to the level of the papillary muscles by
pulsed-wave Doppler. The pressure half-time of the continuous-wave Doppler
tracing of the aortic insufficiency jet is 280 m/sec. The jet height to left ventricular
outflow tract height ratio is 53%. The severity of the aortic insufficiency in this case
is:
A. Physiologic insufficiency

500 Question Study by Terry Reynolds, BS, RDCS


B. Mild (1+)
C. Moderate (2+)
D. Moderately severe (3+)
221. The most common etiology of tricuspid valve stenosis is:
A.
B.
C.
D.

Carcinoid heart disease


Infective endocarditis
Rheumatic fever
Right atrial myxoma

222. The M-mode findings for tricuspid valve stenosis include all the following
EXCEPT:
A.
B.
C.
D.

Anterior motion of the posterior tricuspid valve leaflet


Decreased E-F slope of the anterior tricuspid valve leaflet
Increased leaflet thickness
Systolic doming of the anterior tricuspid valve leaflet

223. The typical two-dimensional echocardiographic findings in rheumatic tricuspid


stenosis include all the following EXCEPT:
A.
B.
C.
D.

Diastolic doming of the anterior tricuspid valve leaflet


Leaflet thickening
Restricted motion of the tricuspid leaflets
Systolic bowing of the posterior tricuspid valve leaflet

224. Causes of tricuspid regurgitation include all the following EXCEPT:


A.
B.
C.
D.

Carcinoid heart disease


Ebsteins anomaly
Pulmonary hypertension
Sinus of Valsalva aneurysm

225. Signs and symptoms of significant tricuspid regurgitation include all the
following EXCEPT:
A.
B.
C.
D.

Hepatomegaly
Jugular venous distention
Pulsus paradoxus
Right ventricular failure

226. The murmur of tricuspid regurgitation is best described as a:


A. Holodiastolic murmur heard best at the lower left sternal border

500 Question Study by Terry Reynolds, BS, RDCS


B. Pansystolic murmur heard best at the lower left sterna border
C. Pansystolic murmur heard best at the cardiac apex with radiation to the axilla
D. Systolic ejection murmur heard best at the upper right sterna border
227. Causes of organic tricuspid regurgitation include all the following EXCEPT:
A.
B.
C.
D.

Rheumatic heart disease


Right ventricular infarct
Tricuspid valve prolapsed
Flail tricuspid valve

228. In significant chronic tricuspid valve regurgitation, all the following are dilated
EXCEPT:
A.
B.
C.
D.

Hepatic veins
Inferior vena cava
Pulmonary veins
Right atrium

229. The M-mode finding for ruptured chordae tendineae of the tricuspid valve is:
A. Coarse diastolic flutter of the anterior tricuspid valve leaflet
B. Fine diastolic flutter of the anterior tricuspid valve leaflet
C. Irregular low-frequency diastolic fluttering of the anterior tricuspid valve
leaflet
D. Right atrial enlargement
230. On M-mode echocardiographic finding of the tricuspid valve, systolic coarse
chaotic oscillation of the tricuspid valve leaflets may indicate:
A.
B.
C.
D.

A normal echocardiographic finding


Atrial fibrillation/atrial flutter
Flail tricuspid valve leaflet
Pulmonic valve insufficiency

231. M-mode and two-dimensional echocardiograophic findings for chronic tricuspid


regurgitation include:
A.
B.
C.
D.

Left ventricular volume overload


Paradoxical septal motion
Protected right ventricle
Right ventricular hypertrophy

232. Methods for determining the severity of tricuspid regurgitation with pulsedwave Doppler include all the following EXCEPT:
A. Increased E wave velocity for the tricuspid valve
B. Mapping technique

500 Question Study by Terry Reynolds, BS, RDCS


C. Maximum velocity of the tricuspid regurgitant jet
D. Systolic flow reversal in the hepatic vein
233. Cardiac Doppler findings associated with significant chronic tricuspid
regurgitation include all the following EXCEPT:
A.
B.
C.
D.

Concave late systolic configuration of the regurgitant signal


Increased E velocity of the tricuspid valve
Systolic flow reversal in the hepatic vein
Systolic flow reversal in the pulmonary vein

234. Systolic reflux of saline contrast noted by two-dimensional echocardiography in


the inferior vena cava denotes the presence of:
A.
B.
C.
D.

Cardiac tamponade
Constrictive pericarditis
Pulmonary insufficiency
Tricuspid regurgitation

235. An intracardiac pressure may be determined from the continuous-wave


tricuspid regurgitation signal is:
A.
B.
C.
D.

Mean pulmonary artery pressure


Pulmonary artery end-diastolic pressure
Systolic pulmonary artery pressure
Total pulmonary vascular resistance

236. Tricuspid valve leaflets that are in a fixed semi-open position with diffuse
thickening are found in:
A.
B.
C.
D.

Carcinoid heart disease


Cardiac amyloidosis
Cardiac hemochromatosis
Cardiac sarcoidosis

237. Possible echocardiographic/Doppler findings in a patient with carcinoid heart


disease include all the following EXCEPT:
A.
B.
C.
D.

Pulmonic valve insufficiency


Pulmonic valve stenosis
Tricuspid valve prolapse
Tricuspid valve regurgitation

238. The most common cause of pathologic pulmonary insufficiency is:

500 Question Study by Terry Reynolds, BS, RDCS


A.
B.
C.
D.

Carciniod heart disease


Infective endocarditis
Pulmonary hypertension
Rheumatic heart disease

239. Pulmonary insufficiency is associated with:


A.
B.
C.
D.

Left ventricular volume overload


Right atrial hypertrophy
Right ventricular hypertrophy
Right ventricular volume overload

240. When pulmonary artery systolic pressure exceeds 70 mmHg, dilation of the
pulmonic annulus results in a regurgitation jet of high velocity which is responsible
for the murmur called:
A.
B.
C.
D.

Austin Flint
Rivero-Carvallo
Graham Steell
Lillehei-Kaster

241. An end-diastolic velocity of 2 m/sec for pulmonary insufficiency was obtained


with an estimated right atrial pressure of 7 mmHg. The pulmonary artery enddiastolic pressure (PAEDP) is:
A.
B.
C.
D.

2 mmHg
16 mmHg
23 mmHg
26 mmHg

242. The normal pulmonary artery end-diastolic pressure (PAEDP) is:


A.
B.
C.
D.

0 to 5 mmHg
4 to 12 mmHg
9 to 18 mmHg
18 to 25 mmHg

243. Fine diastolic flutter of the tricuspid valve is a characteristic finding for:
A.
B.
C.
D.

Infundibular stenosis
Primary pulmonary hypertension
Pulmonary insufficiency
Pulmonary valve stenosis

244. The most common type of right ventricular outflow tract obstruction is:
A. Subinfundibular
B. Subvalvular

500 Question Study by Terry Reynolds, BS, RDCS


C. Supravalvular
D. Valvular
245. The most common etiology of pulmonic valve stenosis is:
A.
B.
C.
D.

Carcinoid
Congenital
Infective endocarditis
Rheumatic

246. The characteristic M-mode pulmonic valve leaflet pattern in pulmonic valve
stenosis is:
A.
B.
C.
D.

Absent a dip
Deep a dip
Reversed a dip
Shallow a dip

247. On M-mode echocardiography, the effect of infundibular pulmonic stenosis on


the pulmonic valve is:
A.
B.
C.
D.

Absent a dip
Coarse systolic flutter
Deep a dip
Shallow a dip

248. Possible two-dimensional echocardiographic findings for valvular pulmonic


stenosis include all the following EXCEPT:
A.
B.
C.
D.

Pulmonic valve prolapsed


Right ventricular hypertrophy
Systolic doming of the pulmonic valve leaflets
Valvular thickening

249. Post-stenotic dilatation of the main pulmonary artery is a two-dimensional


echocardiographic finding for:
A.
B.
C.
D.

Pulmonary insufficiency
Pulmonary turmor
Tricuspid regurgitation
Valvular pulmonic stenosis

250. The right ventricular outflow tract obstruction associated with poststenotic
dilatation of the main pulmonary artery is:
A. Valular
B. Subvalvular
C. Supravalvular

500 Question Study by Terry Reynolds, BS, RDCS


D. Subinfundibular
251. Pulmonary artery banding may result in all the following EXCEPT:
A.
B.
C.
D.

Pseudoaneurysm formation
Right ventricular hypertrophy
Supravalvular pulmonary stenosis
Valvular pulmonic stenosis

252. The most common symptom of infective endocarditis is:


A.
B.
C.
D.

Chest pain
Dyspnea
Fever
Orthopnea

253. The complications of infective endocarditis include all the following EXCEPT:
A.
B.
C.
D.

Congestive heart failure


Embolization
Valve ring abscess
Valvular prolapsed

254. The classic manifestation of infective endocarditis is cardiac valve:


A.
B.
C.
D.

Doming
Sclerosis
Tumor
Vegetation

255. Infective endocarditis is a greater risk in patients with:


A.
B.
C.
D.

Atrial fibrillation
Coronary artery disease
Left ventricular aneurysm
Prosthetic heart valve

256. A patient with a history of mitral valve prolapsed presents to the


echocardiography laboratory with the complaints of fever, night sweats, and weight
loss. The most likely explanation is:
A.
B.
C.
D.

Congestive heart failure


Coronary artery disease
Infective endocarditis
Kawasaki disease

257. The mitral valve is considered to be prematurely closed due to severe acute
insufficiency when the C point of mitral valve closure occurs:

500 Question Study by Terry Reynolds, BS, RDCS


A.
B.
C.
D.

On or before inception of the Q wave


Less than 0.05 seconds after the Q wave
0.05 to 0.07 seconds after the Q wave of the electrocardiogram
On or before inception of the T wave

258. The usual site of attachment for vegetations on the mitral and tricuspid valves
is the:
A.
B.
C.
D.

Annulus
Atrial side of the valve leaflets
Papillary muscles
Ventricular side of valve leaflets

259. The vegetation diameter as determined by two-dimensional echocardiography


that is most often associated with systemic emboli is:
A.
B.
C.
D.

3 mm
5 mm
7 mm
10 mm

260. The essential two-dimensional echocardiographic finding of valve ring abscess


secondary to infective endocarditis may be best described as:
A.
B.
C.
D.

Echolucent
Hyperechoic
Pendunculated
Sessile

261. Aortic ring abscess is usually caused by:


A.
B.
C.
D.

Infective endocarditis
Rheumatic fever
Valvular prolapsed
Valvular regurgitation

262. Complications associated with infective endocarditis that may be indications


for surgery include all the following EXCEPT:
A.
B.
C.
D.

Congestive heart failure


Dyspnea
Myocardial or annular abscess
Recurrent systemic emboli

263. The M-mode appearance of mitral valve and aortic valve vegetations is
described as:
A. Doming
B. Prolapsing
C. Shaggy

500 Question Study by Terry Reynolds, BS, RDCS


D. Relapsing
264. Another term for a hemograft prosthetic valve is:
A.
B.
C.
D.

Allograft
Autograft
Biograft
Heterograft

265. All the following are porcine tissue prosthetic valves EXCEPT:
A.
B.
C.
D.

Bjork-Shiley
Carpentier-Edwards
Hancock
Intact

266. An example of bovine pericardium tissue valve is:


A.
B.
C.
D.

Bjork-Shiley
Intact
Ionescu-Shiley
Starr-Edwards

267. A pulmonic valve relocated to the aortic position is called a(n):


A.
B.
C.
D.

Allograft
Autograft
Heterograft
Homograft

268. Coarse fluttering of a tissue prosthetic valve leaflet on M-mode is associated


with:
A.
B.
C.
D.

Insufficiency
Normal function
Stenosis
Vegetation

269. Abnormal rocking motion of a prosthetic valve by two-dimensional


echocardiography indicates prosthetic valve:
A.
B.
C.
D.

Dehiscence
Stenosis
Thrombus
Vegetation

270. Prosthetic valve prolapsed as seen by two-dimensional echocardiography is


associated with prosthetic valve:

500 Question Study by Terry Reynolds, BS, RDCS


A.
B.
C.
D.

Insufficiency
Normal function
Stenosis
Vegetation

271. A shortened interval between the aortic second sound (A2) and mitral valve
opening recorded for a mitral valve prosthesis may indicate all the following
EXCEPT:
A.
B.
C.
D.

Perivalvular leak
Poor left ventricular function
Prolonged PR interval on the electrocardiogram
Prosthetic mitral valve dysfunction

272. Bioprosthetic leaflet degeneration and calcification occurs:


A.
B.
C.
D.

Only in adults
More frequently in children
Equally in children and adults
Primarily in women 40 years of age or older

273. A prosthetic heart valve is associated with a relatively high rate of outlet strut
fracture is:
A.
B.
C.
D.

Bjork-Shiley
Carpentier-Edwards
Ionescu Shiley
Starr-Edwards

274. An example of a bileaflet tilting disc prosthetic heart valve is:


A.
B.
C.
D.

Medtronic-Hall
Omniscience
St. Judes
Starr-Edwards

275. The cardiac Doppler formula that accurately determines the pressure gradient
in the prosthetic aortic valve is:
A.
B.
C.
D.

4 x (V22)
4 x (V12 V22)
4 x (V22 V12)
Area x V1

276. The best Doppler formula for calculating the effective orifice area (EOA) in a
patient with mitral valve replacement is:
A. 4 x (V2) 2

500 Question Study by Terry Reynolds, BS, RDCS


B. 4 x (V2 2 V12)
C. (CSA LVOT x TVI LVOT) TVI MV
D. 220 pressure half-time
277. The best Doppler method for evaluating an aortic valve replacement is
probably:
A.
B.
C.
D.

Deceleration slope
Maximum peak instantaneous gradient
Pressure half-time
Velocity ratio

278. Doppler evaluation of a prosthetic mitral valve should include all the following
EXCEPT:
A.
B.
C.
D.

Effective orifice area (continuity equation)


Peak A velocity
Peak and mean pressure gradients
Pressure half-time

279. Complications associated with prosthetic heart valve dysfunction include all
the following EXCEPT:
A.
B.
C.
D.

Dehiscence
Leaflet degeneration
Thrombosis
Tumor

280. A regurgitant jet area <1.0 cm2 is noted in a prosthetic aortic valve. This can
be explained by:
A.
B.
C.
D.

Ball variance
Closing volume
Disc embolization
Disc occlusion

281. With thromotic obstruction of a prosthetic valve in the aortic position,


continuous-wave Doppler findings include:
A.
B.
C.
D.

Effective orifice area is normal


Velocity ratio is increased
Maximum velocity is increased
Pressure half-time is increased

282. Clinical evidence of porcine valve dysfunction is most likely to be seen when
leaflets are thicker than:
A. 1 mm

500 Question Study by Terry Reynolds, BS, RDCS


B. 2 mm
C. 3 mm
D. 4 mm
283. The most common etiology of constrictive pericarditis is:
A.
B.
C.
D.

Cardiac surgery
Idiopathic
Rheumatic fever
Tuberculosis

284. The classic auscultatory finding in constrictive pericarditis is:


A.
B.
C.
D.

Friction rub
Mid-systolic click
Opening snap
Pericardial Knock

285. Cardiac catheterization findings in constrictive pericarditis include:


A.
B.
C.
D.

Absent a wave
Dip-and-plateau waveform
Increased v wave
Increased peak-to-peak pressure gradient

286. The square root is commonly found in:


A.
B.
C.
D.

Aortic valve stenosis


Constrictive pericarditis
Pericardial effusion
Pulmonary hypertension

287. Echocardiographic signs associated with constrictive pericarditis include all the
following EXCEPT:
A.
B.
C.
D.

B notch
Inferior vena cava plethora
Railroad track sign
Septal bounce

288. Doppler evidence of constrictive pericarditis includes:


A.
B.
C.
D.

Increased
Increased
Increased
Increased

peak
peak
peak
peak

velocity
velocity
velocity
velocity

across
across
across
across

the
the
the
the

mitral valve with inspiration


aortic valve with inspiration
mitral valve with expiration
tricuspid valve with expiration

289. An M-mode echocardiographic sign of constrictive pericarditis is:

500 Question Study by Terry Reynolds, BS, RDCS


A.
B.
C.
D.

B notch sign
Mid-late systolic dip sign
Smoke-signal sign
Square root sign

290. An M-mode echocardiographic sign for constrictive pericarditis is:


A.
B.
C.
D.

B notch
Chaotic notch
Fibrillatory notch
Spanish notch

291. A possible M-mode finding for constrictive pericarditis is premature opening of


the:
A.
B.
C.
D.

Aortic valve
Mitral valve
Pulmonic valve
Tricuspid valve

292. The absence of inferior vena cava collapse upon inspiration indicates elevated
pressure in the:
A.
B.
C.
D.

Aorta
Left atrium
Left ventricle
Right atrium

293. Low voltage of the QRS complex throughout the electrocardiogram is often
found in:
A.
B.
C.
D.

Constrictive pericarditis
Mitral stenosis
Pericardial effusion
Pleural effusion

294. In acute pericarditis, a possible electrocardiographic finding in most if not all


leads is:
A.
B.
C.
D.

Depressed ST segments
Elevated ST segments
Increased QRS voltage
Pathologic Q waves

295. A possible auscultatory finding in a patient with pericarditis is:

500 Question Study by Terry Reynolds, BS, RDCS


A.
B.
C.
D.

Fixed splitting of S2
Mid-systolic click
Friction rub
Pericardial knock

296. The pulse associated with cardiac tamponade is:


A.
B.
C.
D.

Pulsus
Pulsus
Pulsus
Pulsus

alternans
bisferiens
paradoxus
parvus

297. The single most reliable echocardiographic predictor of cardiac tamponade


that can be identified by M-mode or two-dimensional imaging is diastolic collapse of
the:
A.
B.
C.
D.

Left atrium
Left ventricle
Right atrium
Right ventricle

298. Pulsed-wave Doppler evidence of cardiac tamponade includes:


A. Systolic flow reversal in the pulmonary veins
B. Systolic flow reversal in the hepatic veins
C. Insipiratory increase in peak velocity across the mitral valve with an
inspiratory decrease in peak velocity across the tricuspid valve
D. Inspiratory decrease in velocity across the mitral valve with an inspiratory
increase in velocity across the tricuspid valve
299. The swinging heart syndrome is associated with:
A.
B.
C.
D.

Cardiac trauma
Constrictive pericarditis
Mitral valve prolapsed
Pericardial effusion

300. Diastolic collapse of the right ventricle in cardiac tamponade occurs during:
A.
B.
C.
D.

Early diastole
Mid-diastole
Late diastole
Atrial systole

301. The most effective treatment for cardiac tamponade is:


A. Aspirin

500 Question Study by Terry Reynolds, BS, RDCS


B. Bed rest
C. Pericadiectomy
D. Pericardiocentesis

302. Doppler evidence of cardiac tamponade from diastolic hepatic vein flow is:
A.
B.
C.
D.

Expiratory decrease
Expiratory increase
Inspiratory increase
Inspiratory reversal

303. Fibrin within a pericardial effusion most likely indicates:


A.
B.
C.
D.

Cardiac tamponade
Constrictive pericarditis
Infective endocarditis
Long-standing pericardial effusion

304. In patients with pericardial effusion, an echo-free space will be seen between
the epicardium and the:
A.
B.
C.
D.

Endocardium
Fibrous pericardium
Myocardium
Parietal Serous Pericardium

305: A large pericardial effusion precludes the diagnosis of all the following EXCEPT:
A.
B.
C.
D.

Flail mitral valve


Mitral valve prolapse
Pulmonic valve prolapsed
Systolic anterior motion (SAM) of the mitral valve

306. A posterior echo-free space is detected during the systolic phase only by Mmode/two-dimensional echocardiography. This is considered a:
A.
B.
C.
D.

Normal finding
Small pericardial effusion
Moderate pericardial effusion
Large pericardial effusion

307. Air in the pericardial sac is known as:


A.
B.
C.
D.

Cardiac tamponade
Effusive-constrictive pericardium
Hemopericardium
Pneumopericardium

500 Question Study by Terry Reynolds, BS, RDCS

308. The best guideline for differentiating pericardial effusion from pleural effusion
by two-dimensional echocardiography is:
A. Pericardial effusion is located anterior to the descending aorta; pleural
effusion is present posterior to the descending aorta.
B. Pericardial effusion is present posterior to the descending aorta; pleural
effusion is located anterior to the descending aorta.
C. Pericardial effusion is usually seen as an anterior clear space; pleural
effusion is usually seen as a posterior clear space.
D. Pericardial effusion is usually seen as a posterior clear space; pleural
effusion is usually seen as an anterior clear space.
309. The most common location for a pericardial cyst is the:
A.
B.
C.
D.

A.Hilium
B.Left costophrenic angle
Right costophrenic angle
Superior mediastinum

310. An echocardiographic finding in congenital absent pericardium is volume


overload of the:
A.
B.
C.
D.

Left atrium
Left ventricle
Right atrium
Right ventricle

311. The most common etiology of systemic hypertension is:


A.
B.
C.
D.

A.Idiopathic processes
Renal disease
Pheochromocytoma
Psychogenic origin

312. Secondary findings associated with systemic hypertension include al the


following EXCEPT:
A.
B.
C.
D.

Increased left ventricular mass


Increased right ventricular mass
Left atrial enlargement
Left ventricular hypertrophy

313. A patient with chronic systemic hypertension presents to the


echocardiography laboratory. The following pulsed-wave Doppler data is acquired
from the tips of the mitral valve leaflets: E:A ratio 1.2:1, deceleration time 210
msec, isovolumic relaxation time 83 msec, and pulmonary vein reversal 46cm/sec.
The Doppler data indicates diastolic filling stage:
A. I
B. II
C. III

500 Question Study by Terry Reynolds, BS, RDCS


D. IV

314. Electrocardiographic evidence associated with systemic hypertension


includes:
A.
B.
C.
D.

Left atrial enlargement


Left ventricular hypertrophy
Right atrial enlargement
Right ventricular hypertrophy

315. The most reliable M-mode indicator for pulmonary hypertension is:
A.
B.
C.
D.

Deep a wave of the pulmonic valve


Mid-systolic notching of the pulmonic valve
Shallow a dip of the pulmonic valve
Systolic flutter of the pulmonic valve

316. M-mode findings associated with pulmonary hypertension include:


A.
B.
C.
D.

Absent or shallow a dip of the pulmonic valve


Deep a dip of the pulmonic valve
Paradoxical a dip of the pulmonic valve
Reverse a dip of the pulmonic valve

317: The most of the interventricular septum in pulmonary hypertension is:


A.
B.
C.
D.

Akinetic
Hyperkinetic
Hypokinetic
Paradoxical

318: A two-dimensional echocardiographic finding associated with pulmonary


hypertension is:
A.
B.
C.
D.

Flattening of the interventricular septum in diastole


Flattening of the interventricular septum in systole
Dyskinetic interventricular septal motion
Hyperkinetic interventricular septal motion

319. The Doppler finding used to calculate mean pulmonary artery pressure is:
A.
B.
C.
D.

Mitral regurgitation
Pulmonary insufficiency
Right ventricular outflow tract acceleration time
Tricuspid regurgitation

320. All the following may be used to calculate pulmonary artery pressure b cardiac
Doppler EXCEPT:
A. Mitral regurgitation
B. Pulmonary insufficiency

500 Question Study by Terry Reynolds, BS, RDCS


C. Right ventricular outflow tract acceleration
D. Tricuspid regurgitation

321. Possible echocardiographic findings for pulmonary hypertension include all the
following EXCEPT:
A.
A.
B.
C.

Right atrial enlargement


B.Right ventricular enlargement
Pulmonary vein enlargement
Tricuspid regurgitation

322. The primary pulsed-wave mitral valve Doppler diastolic abnormality in a


patient with systemic hypertension is stage:
A.
B.
C.
D.

I
II
III
IV

323. A more appropriate name for idiopathic hypertrophic subaortic stenosis (IHSS)
is:
A.
B.
C.
D.

Aortic tunnel disease (ATD)


Discrete subaortic valve stenosis (DSS)
Hypertrophic Cardiomyopathy (HCM)
Subaortic hourglass deformity (SHD)

324. A common electrocardiographic finding in patients with hypertrophic


obstructive cardiomyopathy is:
A.
B.
C.
D.

Depressed ST segments
Elevated ST segments
Left ventricular hyoertrophy
Right ventricular hypertrophy

325. With M-mode echocardiography, asymmetric septal hypertrophy is present


when the interventricular septum to posterior wall ratio is:
A.
B.
C.
D.

greater
greater
greater
greater

than
than
than
than

or
or
or
or

equal
equal
equal
equal

to
to
to
to

0:1
1:1
1.2:1
1.3:1

326. The mitral valve finding most strongly associated with hypertrophic
obstructive cardiomyopathy is mitral valve:
A.
B.
C.
D.

Aneurism
Fenestration
Flail leaflet
Systolic anterior motion

500 Question Study by Terry Reynolds, BS, RDCS

327. A hallmark M-mode aortic valve finding in patients with hyper trophic
obstructive cardiomyopathy is aortic valve:
A.
B.
C.
D.

Diastolic flutter
Fenestration
Mid-systolic notching
Vegetation

328. A systolic high-velocity, late-peaking, dagger-shaped, continuous-wave


Doppler signal is obtained. The most likely diagnosis is:
A.
B.
C.
D.

Hypertrophic obstructive cardiomyopathy


Mitral regurgitation
Tricuspid regurgitation
Valvular aortic stenosis

329. M-mode findings associated with hypertrophic cardiomyopathy include all the
following EXCEPT:
A.
B.
C.
D.

Asymmetric septal hypertrophy


Mid-systolic notching of the aortic valve
Mid-systolic notching of the pulmonary valve
Systolic anterior motion of the mitral valve

330. Common two-dimensional echocardiographic findings in hypertrophic


obstructive cardiomyopathy include all the following EXCEPT:
A.
B.
C.
D.

Endocardial plaqueing of the interventricular septum


Left atrial enlargement
Left ventricular enlargement
Thickening of the anterior mitral valve leaflet

331. Pulsed-wave and color flow Doppler are useful in hypertrophic obstructive
cardiomyopathy in all the following ways EXCEPT:
A.
B.
C.
D.

Aid in guiding the continuous- wave Doppler beam


Determine the presence and severity and severity of mitral regurgitation
Help distinguish left ventricular outflow tract flow from mitral regurgitation
Quantitate the severity of the left ventricular outflow tract obstruction

332. The pulsed-wave Doppler mitral flow pattern most often associated with
hypertrophic obstructive cardiomyopathy is stage:
A.
B.
C.
D.

I
II
III
IV

500 Question Study by Terry Reynolds, BS, RDCS

333. A speckled or ground-glass appearance of the ventricular septum seen twodimensional echocardiography is found in:
A.
B.
C.
D.

Constrictive Pericarditis
Coronary artery disease
Dilated cardiomyopathy
Hypertrophic cardiomyopathy

334. Characteristics findings in patients with idiopathic dilated cardiomyopathy


include all the following EXCEPT:
A.
B.
C.
D.

Assymetric septal hypertrophy


Dilated, poorly contracting left ventricle
Low cardiac output
High intracardiac pressures

335. The characteristics shape of the left ventricular in patients with dilated
cardiomyopathy is:
A.
B.
C.
D.

Elongated
Rectangular
Spherical
Triangular

336. Early in the disease stage, the usual Doppler mitral inflow pattern in patients
with dilated cardiomyopathy demonstrates:
A.
B.
C.
D.

Abnormal compliance pattern


Abnormal relaxation pattern
Normal pattern
Pseudonormal pattern

337. A common mitral valve finding of dilated cadiomyopathy in two-dimensional


echocardiography is:
A.
B.
C.
D.

Decreased E septal separation


Reverse diastolic doming
Incomplete closure of the mitral valve
Premature closure of the mitral valve

338. Characteristics signs of decreased stroke volume on M-mode include all the
following EXCEPT:
A.
B.
C.
D.

Decreased D-E excursion of the mitral valve


Gradual closure of the aortic valve during systole
Increased a dip of the pulmonic valve
Increased mitral valve E-point septal separation

339. The most common regurgitation found in patients with dilated cardiomyopathy
is:

500 Question Study by Terry Reynolds, BS, RDCS


A.
B.
C.
D.

Aortic insufficiency
MR
Pulmonary insufficiency
Tricuspid regurgitation

340. The most common etiology of secondary, non-ischemic dilated


cardiomyopathy in the Western world is:
A.
B.
C.
D.

Alcohol
Infection (e.g., viral)
Chemotherapy
Pregnancy (postpartum)

341. Functional classifications of cardiomyopathy include all the following EXCEPT:


A.
B.
C.
D.

Dilated (congestive)
Hypertrophic
Non-dilated
Restrictive

342. A common late complication associated with dilated cardiomyopathy is:


A.
B.
C.
D.

Infective endocarditis
MR
Systemic emboli
Ventricular gallops

343. Echocardiographic findings in dilated cardiomyopathy include all the following


EXCEPT:
A.
B.
C.
D.

Apical mural thrombus


Dilated ventricular cavities
Enlarged atrial cavities
Increased mitral valve leaflet excursion

344. The most common cause of primary dilated cardiomyopathy is:


A.
B.
C.
D.

Adriamyacin toxicity
CAD
Hemochromatosis
Idiopathic

345. Possible causes of restrictive cardiomyopathy include all the following EXCEPT:
A.
B.
C.
D.

Alcohol
Amyloidosis
Hemochromatosis
Sarcoidosis

346. The echocardiographic features of amyloidosis include all the following


EXCEPT:
A. Dilatation of the ascending aorta

500 Question Study by Terry Reynolds, BS, RDCS


B. Increased ventricular wall thickness
C. Multivalvular regurgitation
D. Pericardial effusion

347. The cardiomyopathy with which cardiac hemochromatosis is most often


associated is:
A.
B.
C.
D.

Dilated
Hypertrophic
Idiopathic
Subaortic

348. Possible echocardiographic findings in sarcoid heart disease include:


A.
B.
C.
D.

Assymetric septal hypertrophy


Concentric left ventricular hypertrophy
Dilatation of the ascending aorta
Posterobasal aneurism

349. Common echocardiographic/Doppler findings in systemic lupus erythematosus


include:
A.
B.
C.
D.

Aortic dissection
Granular appearance of the myocardium
Pericarditis
Valvular aortic stenosis

350. The cardiac involvement associated with acquired immunodeficiency


syndrome (AIDS) is:
A.
B.
C.
D.

Dilated Cardiomyopathy
Hypertrophic cardiomyopathy
Infiltrative cardiomyopathy
Restrictive cardiomyopathy

351. The most common etiology for ischemic heart disease is coronary artery:
A.
B.
C.
D.

Aneurysm
Atherosclerosis
Embolus
Spasm

352. Angina occurring at rest that is not preceded by exercise or an increase in


heart rate is called:
A.
B.
C.
D.

Labile
Prinzmetals
Stable
Unstable

353. The most specific echocadiographic findings for ischemic muscle is:

500 Question Study by Terry Reynolds, BS, RDCS


A.
B.
C.
D.

Abnormal diastolic wall motion at the ischemic segment


Alterations In systolic thickening
Normal diastolic wall motion
Normal systolic wall motion

354. Tardokinesis is a delay in:


A.
B.
C.
D.

Left ventricular filling


Right ventricular filling
Valve opening
Wall motion

355. A wall motion score of 3 assigned to a certain segment of left ventricular


muscle indicates:
A.
B.
C.
D.

Akinetic wall motion


Dyskinetic wall motion
Hypokinetic wall motion
Normal wall motion

356. The definition of stunned myocardium is:


A.
B.
C.
D.

Myocardium after cardiopulmonary resuscitation


Myocardium after electrical cardioversion
Myocardium that is hyperkinetic post-myocardial infarction
Reperfused viable myocardium that is not functioning

357. Hibernating myocardium is:


A.
B.
C.
D.

Myocardium that is hyperkinetic post-myocardial infarction


Reperfused viable myocardium that is not functioning
Viable myocardium at rest but not functioning with exercise
Viable myocardium that is nonfunctioning because of chronic ischemia

358. Stress echocardiography methods may be used to detect stunned or


hibernating myocardium include:
A.
B.
C.
D.

Cold pressure
Handgrip
Low-dose dobutamine
Treadmill

359. In determining the size of myocardial infarction, echocardiography generally:


A. Is unpredictable
B. Overestimates recent myocardial infarction and underestimates old
myocardial infarction
C. Predicts the exact size of infarct
D. Underestimates recent myocardial infarction and overestimates old
myocardial infarction

500 Question Study by Terry Reynolds, BS, RDCS


360. The normal response of non-infarcted myocardium in a patient with acute
myocardial infarction is:
A.
B.
C.
D.

A.Akinesis
Dyskinesis
Hyperkinesis
Hypokinesis

361. Coronary artery perfusion occurs from:


A.
B.
C.
D.

Endocardium to epicardium
Epicardium to endocardium
Epicardium to myocardium
Myocardium to endocardium

362. The echocardiographic appearance of neorotic myocardium secondary to


myocardial infarction includes all the following EXCEPT:
A.
B.
C.
D.

Akinetic myocardial wall


Echogenic wall segment
Ground-glass appearance
Thin ventricular wall

363. Echocardiographic findings in the post myocardial infarction patient include:


A.
B.
C.
D.

Mitral annular calcification


Mural thrombus
Valvular Stenosis
Ventricular septal aneurysm

364. The correct term for describing decreased ventricular wall motion is:
A.
B.
C.
D.

Akinetic
Dyskinetic
Hyperkinetic
Hypokinetic

365. The infarction most commonly associated with left ventricular aneurysm is:
A.
B.
C.
D.

Anterior
Inferior
Lateral
True posterior

366. Echocardiography differentiates a psuedoanerysm from a true ventricular


aneurysm by the:
A.
B.
C.
D.

Diastolic motion of the aneurysm


Length of the aneurysm
Width of the border of the aneurysm
Width of the neck of the anerysm

500 Question Study by Terry Reynolds, BS, RDCS


367. An area of diseased myocardium is resected and he remaining healthy tissue
is sutured together. This surgical procedure is called:
A.
B.
C.
D.

Aneurysmectomy
Myectomy
Myotomy
Pericardiectomy

368. Possible mechanisms in the development of mitral regurgitation following an


acute myocardial infarction include all the following EXCEPT:
A.
B.
C.
D.

Fibrosis of the papillary muscle


Incomplete closure of the mitral valve
Mitral valve prolapsed
Papillary muscle rupture

369. A possible etiology for pericardial effusion is:


A.
B.
C.
D.

Acute MI
Chronic AI
Chronic MR
Mitral valve stenosis

370. The expected convetional Doppler finding in a patient with an acquired


ventricular septal defect is:
A. Laminar-flow velocity flow during diastole on the left side of the
interventricular septum
B. Laminar high velocity flow during diastole on the left side of the
interventricular septum
C. Turbulent high velocity flow in diastole on the right side of the septum
D. Turbulent high velocity flow in systole on the right side of the
interventricular septum
371. The type of infarction that most often involves the right ventricle is:
A.
B.
C.
D.

Anterior
Antero-postero
Inferior
Lateral

372. The principle echocardiographic/Doppler abnormalities of right ventricular


infarction include all the following EXCEPT:
A.
B.
C.
D.

Abnormal motion of the right ventricular free wall


Right ventricular dilatation
Right ventricular hypertrophy
TR

373. While examining a patient with Kawasaki disease, the echocardiographer


should be careful to rule out:
A. Aortic root dilatation

500 Question Study by Terry Reynolds, BS, RDCS


B. Coronary artery aneurysm
C. Coronary artery atherosclerosis
D. Mitral valve prolapse

374. Medications that may be used to perform stress echocardiography include all
the following EXCEPT:
A.
B.
C.
D.

Adenosine
Dipyridamole
Dobutamine
Propranolol

375. A wall segment of the heart that is without motion is best described as:
A.
B.
C.
D.

Akinetic
Dyskinetic
Hyperkinetic
Hypokinetic

376. A dumbbell-shaped configuration of the interatrial septum is associated with:


A.
B.
C.
D.

Amyloidosis
Lipomatous hypertrophy
Sarcoidosis
Sarcoma

377. The most common primary benign cardiac neoplasm is:


A.
B.
C.
D.

Fibroma
Lipoma
Myxoma
Papilloma

378. The most common primary benign cardiac tumor found in children is:
A.
B.
C.
D.

Myxoma
Papilloma
Rhabdomyoma
Rhabdomyosarcoma

379. A primary benign cardiac tumor that is found most often on the endocardial
surface of the atrioventicular valves or valvular endocardium is:
A.
B.
C.
D.

Fibroma
Lipoma
Myxoma
Papilloma

500 Question Study by Terry Reynolds, BS, RDCS


380. The most common valvular tumor is the:
A.
B.
C.
D.

Angiosarcoma
Fibroelastoma
Lipoma
Myxoma

381. A thrombus shape that is associated with embolization is:


A.
B.
C.
D.

Spherical
Flat
Eccentric
Pedunculated

382. An unattached, freely moving clot within the left atrium is referred to as a:
A.
B.
C.
D.

Ball thrombus
Pedunculated thrombus
Sessile thrombus
Stationary thrombus

383. Complications of a right atrial clot include:


A.
B.
C.
D.

Interatrial septal aneurysm


Patent foramen ovale
Pulmonary embolism
Systemic embolism

384. The useful artifact associated with lodging of a bullet within the heart is:
A.
B.
C.
D.

Enhancement
Mirroring
Shadowing
Reverberation

385. All the following statements concerning metastases of cardiac tumors are true
EXCEPT:
A. Metastases are 10 to 40 times more likely than primary lesions
B. The most common metastatic tumor is a direct extension of lung and
breast cancer
C. The most common metastatic tumor is myxoma
D. Renal cell carcinoma may present as a right atrial mass by direct
extension up the inferior vena cava .
386. Likely complications of cardiac metastatic tumors include all the following
EXCEPT:
A.
B.
C.
D.

Congestive heart failure due to myocardial infiltration.


Embolization of parts or pieces of the tumor.
Pericardial effusion/tamponade.
Valvular fenestration.

500 Question Study by Terry Reynolds, BS, RDCS


387. All the following are true statements concerning primary cardiac tumors
EXCEPT:
A.
B.
C.
D.

Angiosarcoma is the most common pediatric primary cardiac tumor.


Myxoma is the most common primary cardiac tumor.
Primary intracardiac tumors are often benign (3:1)
Primary pericardial tumors have a 50% chance of being malignant.

388. The cardiac chambers in which rhabdomyomas are most often visualized are
the:
A.
B.
C.
D.

Atria.
Great vessels.
Ventricles.
Atria, great vessels, and ventricles equally.

389. Pericardial tumors include all the following EXCEPT:


A.
B.
C.
D.

Angiosarcoma.
Lipoma.
Mesothelioma.
Rhabdomyosarcoma.

390. The most common primary malignant tumor of the heart is:
A.
B.
C.
D.

Angiosarcoma.
Fibroma.
Lipoma.
Rhabdomyoma.

391. The principal echocardiographic feature of the left bundle branch block is:
A.
B.
C.
D.

Early systolic dip.


Hyerkinesis of the interventricular septum.
Hyperkinesis of the posterior wall of the left ventricle.
Posterior motion of the interventricular septum.

392. Electrical pacing of the right ventricle mimics the electrocardiographic and
echocardiographic findings of:
A.
B.
C.
D.

Complete atrioventricular block.


Left bundle branch block.
Right bundle branch block.
Wolff-Parkinson-White syndrome.

393. Possible echocardiographic findings for patients with right bundle branch block
include:
A. Decreased interval between tricuspid valve closure and pulmonic valve
opening.
B. Early, systolic beaking of the interventricular septum.

500 Question Study by Terry Reynolds, BS, RDCS


C. Increased interval between mitral and tricuspid valve closure.
D. Systolic paradoxical septal motion.
394. Possible echocadiographic findings in patients with Wolff-Parkinson-White
type B syndrome include a(n):
A. Brief anterior displacement of the left ventricular wall.
B. Brief posterior displacement of the right ventricular wall.
C. increase in the interval between mitral valve closure and pulmonic valve
opening.
D. Sharp, brief, systolic downward posterior dip of the interventricular
septum.
395. Possible echocardiographic findings in patients with Wolff-Parkinson-White
type A syndrome include a(n):
A. Brief anterior displacement of the left ventricular wall.
B. Brief posterior displacement of the right ventricular wall.
C. Increase in the interval between mitral valve closure and pulmonic valve
opening.
D. Sharp, brief, downward posterior dip of the interventricular septum.
396. The formula used to determine percent shortening is:
A.
B.
C.
D.

(EDD-ESD) 100.
(EDV-ESV) 100.
(EDD-ESD) EDD 100
(EDV-ESV) EDD100

397. The descent of the mitral annulus in the apical four-chamber view may be
used to evaluate:
A.
B.
C.
D.

Global left ventricular systolic function


Segmental left ventricular function
Severity of mitral regurgitation
Severity of aortic valve stenosis

398. The rate at which the left ventricular pressure rises in systole is referred to as:
A.
B.
C.
D.

dv/dt
dP/dt
dt/dP
dd/tP

399. A B notch of the mitral valve on M-mode indicates increased left ventricular:
A.
B.
C.
D.

End-diastolic pressure
End-systolic pressure
Mean pressure
Peak-to-peak pressure

400. An increased mitral E-point to septal separation may indicate left ventricular:

500 Question Study by Terry Reynolds, BS, RDCS


A.
B.
C.
D.

Decrease in compliance
Decrease in ejection fraction
Hyperdynamic wall motion
Increase in end-diastolic pressure

401. When compared with angiographic volumes, echocardiographic ventricular


volumes are:
A.
B.
C.
D.

A.Equal
Larger
Smaller
Variable, depending on the method used to determine echocardiographic
volume

402. A pulsed-wave Doppler tracing of the mitral valve inflow is obtained with the
following information: E:A ratio is 0.7:1, deceleration time is 320 msec, isovolumic
relaxation time is 110 msec, and pulmonary vein a wave is 22 cm/sec. These
findings are most consistent with:
A.
B.
C.
D.

Normal diastolic function


Stage I diastolic filling pattern
Stage II diastolic filling function
Stage III diastolic filling pattern

403.A pulsed-wave Doppler tracing of the mitral valve inflow is obtained with the
following information: E:A RATIO IS 2.3:1, deceleration time is 110 msec, isovolumic
relaxation time is 52 msec, and pulmonary vein a wave reversal is 44 cm/sec.
These findings are consistent with:
A.
B.
C.
D.

Normal left ventricular diastolic filling


Stage I diastolic filling pattern
Stage II diastolic filling pattern
Stage III diastolic filling pattern

404. A common echocardiographic postoperative finding in a cardiac surgery


patient is:
A.
B.
C.
D.

Paradoxical septal motion


Pleural effusion
Valvular prolapsed
Valvular stenosis

405. The two-dimensional echocardiographic finding in acute pulmonary embolism


is:
A.
B.
C.
D.

Left ventricular dilatation


Left ventricular hypertrophy
Right ventricular dilatation
Right ventricular hypertrophy

406. A common two-dimensional echocardiographic finding in patients with chronic


renal failure is:

500 Question Study by Terry Reynolds, BS, RDCS


A.
B.
C.
D.

Pericardial effusion
Pulmonary hypertension
Valvular regurgitation
Valvular stenosis

407. A left atrial dimension of 68 mm is obtained by M-mode echocardiography.


This measurement is best described as:
A.
B.
C.
D.

Normal
Mildly dilated
Moderately dilated
Severely dilated

408. The left atrial dimension can be measured in the apical four-chamber view
from the mitral ring to the posterosuperior roof at end-systole. In normal subjects,
the upper limit of normal is:
A.
B.
C.
D.

2.6
3.6
4.2
5.2

0.6
0.6
0.6
0.6

cm
cm
cm
cm

409. Findings associated with congenital aneurysm of the left atrium include:
A.
B.
C.
D.

Electrical alternans
Cardiac arrhythmia
Bundle branch block
Pathological Q wave

410. Dilated coronary sinus has been associated with all the following EXCEPT:
A.
B.
C.
D.

Coronary atrioventricular fistula with drainage into the coronary sinus


MR
Persistent left superior vena cava
Right atrial hypertension

411. Right coronary aortic valve leaflet prolapsed will most likely be seen in a
patient with:
A.
B.
C.
D.

Coarctation of the aorta


Membranous ventricular septal defect
Ostium primum atrial septal defect
Patent ductus arteriosus

412. The view of choice when examining a patient with scundum atrial septal
defect is:
A. Apical four chamber
B. Parasternal long axis

500 Question Study by Terry Reynolds, BS, RDCS


C. Parasternal short axis view of the aortic valve
D. Subcostal four chamber
413. An unexpected postoperative finding with atrial septal defect repair is:
A.
B.
C.
D.

Atrial septal aneurysm


Mitral valve prolapsed
Paradoxical interventricular septal motion
Right ventricular volume overload pattern

414. A complete atrioventricular canal defect is ostium primum atrial septal defect
with:
A.
B.
C.
D.

Coarctation
Cleft mitral valve, coarctation of the aorta
Canal-type ventricular septal defect, patent ductus arteriosus
Canal-type ventricular septal defect, common atriovetricular valve

415. The congenital heart defect most often associated with Downs syndrome
(trisomy 21) is:
A.
B.
C.
D.

Coarctation of the aorta


Endocardial cushion defect
Peripheral pulmonary stenosis
Tetralogy of Fallot

416. The pulsed-wave Doppler characteristics of an uncomplicated atrial septal


defect are:
A.
B.
C.
D.

Low velocity left-to-right flow.


High velocity left-to- right flow.
Low-velocity right-to-left flow.
High-velocity right-to-left flow

417. The most common type of atrial septal defect is:


A.
B.
C.
D.

Coronary sinus.
Ostium primum
Ostium secundum
Sinus venosus.

418. The congenital heart defect most commonly associated with ostium primum
atrial septal defect is:
A.
B.
C.
D.

Cleft mitral valve.


Parachute mitral valve.
Partial anomalous pulmonary venous return
total anomalous pulmonary venous return

500 Question Study by Terry Reynolds, BS, RDCS


419. An oxygen saturation sample taken at the superior vena cava is 74% and in
the right atrium is 88%. A possible explanation is:
A.
B.
C.
D.

Atrial septal defect.


Coarctation of the aorta.
Patent ductus arteriosus .
Ventricular septal defect.

420. The classic M-mode finding for atrial septal defect is:
A.
B.
C.
D.

Left ventricular volume overload.


Right ventricular volume overload.
Left ventricular pressure overload.
Right ventricular pressure overload

421: Types of atrial septal defect include all the following EXCEPT:
A.
B.
C.
D.

Ostium primum.
Ostium secundum.
Perimembranous.
Sinus venosus.

422. Cardiac chambers that are enlarged in atrial septal defect include all the
following EXCEPT:
A.
B.
C.
D.

Left atrium.
Main pulmonary artery.
Right atrium
Right ventricle

423. The most common type of ventricular septal defect is:


A.
B.
C.
D.

Inlet (posterior).
Membranous.
Outlet (supracristal).
Trabecular

424. The type of ventricular septal defect most often associated with ventricular
septal aneurysm is:
A.
B.
C.
D.

Inleft
Outlet.
Perimembranous .
Trabecular.

425. The cardiac chambers that are enlarged in vemtricular septal defect are:
A.
B.
C.
D.

Left atrium and left ventricle.


Right atrium and left atrium
Right atrium and right ventricle.
Right ventricle and left ventricle.

500 Question Study by Terry Reynolds, BS, RDCS


426. A congenital malformation of the tricuspid valve in which one, two, or all three
leaflets are displaced downward from the annulus is known as:
A.
B.
C.
D.

Ebsteins anomaly
Epstein-Barr anomaly.
Tricuspid atresia.
Tricuspid valve stenosis.

427. The best echocardiographic view for determining the presence of Ebsteins
anomaly is the:
A.
B.
C.
A.

Apical four-chamber view.


Parasternarnal long-axis view.
Parasternal short-axis view of the aortic valve.
D, Subcostal four-chamber view.

428. Important factors when considering surgical repair in Ebsteins anomaly


include all the following EXCEPT:
A.
B.
C.
D.

Degree of anterior tricuspid valve prolapse.


Degree of atrialization of the right ventricle.
Dysplasia of the valve leaflets.
Extent of leaflet tethering.

429. Congenital heart diseases strongly associated with Ebsteins anomaly include:
A.
B.
C.
D.

Atrial septal defect.


Coarctation of the aorta.
Discrete subaortic stenosis .
Parachute mitral valve.

430. Common cardiac Doppler findings in Ebsteins anomaly include all the
following EXCEPT:
A.
B.
C.
D.

Atrial septal defect with right to-left shunting.


Funtional right ventricular outflow tract obstruction.
Mitral regurgitation.
Tricuspid regurgitation.

431. The classic M-mode finding for Ebsteins anomaly is:


A.
B.
C.
D.

Delayed closure of the tricuspid valve


Flail tricuspid valve leaflet
Tricuspid valve prolapse
Tricuspid valve stenosis

432. Expected echocardiographic findings in patients with Ebsteins anomaly


include all the following EXCEPT:
A.
B.
C.
D.

Abnormal apical insertion of the tricuspid valve


Atrial septal defect
Left ventricular volume overload
Right ventricular volume overload

500 Question Study by Terry Reynolds, BS, RDCS


433. Another name that identifies Uhls anomaly is:
A.
B.
C.
D.

Barlows syndrome
Ebsteins anomaly
Parchment heart
Right ventricular aplasia

434. Persistent patent ductus arteriosus results in:


A.
B.
C.
D.

Right ventricular volume overload


Left ventricular volume overload
Right ventricular pressure overload
Left ventricular pressure overload

435. Persistent patent ductus arteriosus during a pulsed-wave dopper examination


may be confused with all the following EXCEPT:
A.
B.
C.
D.

Pulmonic insufficiency
Tricuspid indufficiency
Aortopulmonary window
Anomalous origin of the left coronary artery from the pulmonary artery

436. The sufficient of a bidirectional persistent ductus arteriosus shunt is that it:
A.
B.
C.
D.

Is an expected (normal) finding


Implies elevated systemic pressure
Implies elevated pulmonary pressure
Negates the simplified Bernoulli equation

437. Congenital heart diseases that are ductal-dependant include all the following
EXCEPT:
A.
B.
C.
D.

Aortic atresia
Interrupted aortic arch
Pulmonary atresia
Sinus venosus atrial septal defect

438. The typical murmur associated with patent ductus arteriosus is:
A.
B.
C.
D.

Continuous murmur
Decreased diastolic murmur
Holosystolic murmur
Late systolic murmur

439. The cardiac chambers that are enlarged in patent ductus arteriosus are:
A.
B.
C.
D.

Left atrium and left ventricle


Left atrium and right ventricle
Right atrium and left ventricle
Right atrium and right ventricle

440. The maximum velocity of a persistent patent ductus arteriosus is 4 m/sec and
the systolic blood pressure is 90/60. The systolic pulmonary artery pressure is:

500 Question Study by Terry Reynolds, BS, RDCS


A.
B.
C.
D.

4 mmHg
26 mmHg
26 mmHg plus right atrial pressure
64 mmHg

441. The Doppler finding associated with persistent patent ductus arteriosus is:
A.
B.
C.
D.

Diastolic flow reversal in the descending thoracic aorta


Increased flow velocity at the aortic isthmus
Increased pressure half-time of the mitral valve
Systolic flow reversal in the pulmonary veins

442. Valvular lesions with which coarctation of the aorta is strongly associated
include:
A.
B.
C.
D.

Aortic valve prolapsed


Aortic valve regurgitation
Bicuspid aortic valve
Pulmonic valve stenosis

443. An underestimated Doppler peak pressure gradient in aortic coarctation may


be caused by:
A.
B.
C.
D.

Patent ductus arteriosus


Significant aortic insufficiency
Significant MR
Ventricular septal defect

444. Narrowing of the aorta at the aortic isthmus is:


A.
B.
C.
D.

Aortic arch aneurysm


Aortic dissection
Coarctation of the aorta
Persistent patent ductus arteriosus

445. In coarctation of the aorta, blood pressure in the legs:


A.
B.
C.
D.

Is lower than in the right arm


Is higher than in the right arm
Is equal to blood pressure in the right arm
Cannot be compared with blood pressure in the right arm

446. Turners syndrome is strongly associated with:


A.
B.
C.
D.

Atrial septal defect


Coarctation of the aorta
Tetralogy of Fallot
Truncus arteriosus

447. The four defects that make up tetralogy of Fallot are pulmonary stenosis,
ventricular septal defect, right ventricular hypertrophy, and:

500 Question Study by Terry Reynolds, BS, RDCS


A.
B.
C.
D.

Atrial septal defect


Cleft mitral valve
Coarctation of the aorta
Deviation of the aorta

448. Defects that are associated with tetralogy of Fallot in about 25% of cases
include:
A.
B.
C.
D.

Bicuspid aortic valve


Overriding pulmonary artery
Parachute tricuspid valve
Right aortic arch

449. Important factors in evaluating post-surgical repair of tetralogy of Fallot


include all the following EXCEPT:
A.
B.
C.
D.

Evaluate right and left ventricular function


Rule out residual pulmonic valve stenosis
Rule out residual shunting at the margins of the atrial septal defect repair
Rule out residual shunting at the margins of the ventricular septal defect
repair

450. Prime characteristics of tetraolgy of Fallot include all the following EXCEPT:
A.
B.
C.
D.

Atrial septal defect


Malalignment ventricular septal defect
Pulmonic stenosis
Right ventricular hypertrophy

451. The anomaly characterized by a single great vessel arising from the base of
the hear t is called:
A.
B.
C.
D.

L-transposition of the great arteries


Pulmonary atresia
Tetralogy of Fallot
Truncus arteriosus

452. A communication between the ascending aorta and the main pulmonary
artery is called:
A.
B.
C.
D.

Aortopulmonary window
Coarctation of the aorta
Patent ductus arteriosus
Supracristal septal defect

453. Possible repairs for D-transposition of the great arteries include all the
following EXCEPT:
A. Blalock-Taussig
B. Jatene
C. Mustard

500 Question Study by Terry Reynolds, BS, RDCS


D. Senning
454. Eisenmengers syndrome may be associated with all the following EXCEPT:
A.
B.
C.
D.

Atrial septal defect


Bicuspid aortic valve
Patent ductus arteriosus
Ventricular septal defect

455. A connective tissue disorder associated with aortic aneurysm is:


A.
B.
C.
D.

Ehlers-Danlos syndrome
Marfans syndrome
Pseudoxanthoma elasticum
Turners syndrome

456. The physical finding of cyanosis is common in:


A.
B.
C.
D.

Atrial septal defect


Eisenmengers syndrome
Patent ductus arteriosus
Ventricular septal defect

457. A possible etiology for aortic aneurysm is:


A.
B.
C.
D.

Aortic insufficiency
Coronary artery disease
Dilated cardiomyopathy
Marfans syndrome

458. The cardiovascular abnormalities seen in patients with Marfans syndrome


include all the following EXCEPT:
A.
B.
C.
D.

Aortic dissection
Dilatation of the aortic root
Mitral valve prolapsed
Myocardial ischemia

459. A fusiform aneurysm is an aneurysm that involves the:


A. One point of the aorta at which there is an expansion of a pouch with a
relatively small neck
B. Descending aorta only
C. Entire aorta from the aortic root to the ligamentum arteriosum
D. Entire circumference of the aorta
460. Echocardiographic criteria for the diagnosis of aortic dissection include all the
following EXCEPT:
A. Decrease in aortic root dimension
B. Normal aortic leaflet motion
C. Recognition of an intimal flap as an oscillating two-dimensional structure
within the aorta

500 Question Study by Terry Reynolds, BS, RDCS


D. Widening of the anterior and posterior aortic root walls
461. An intimal flap in the aorta is discovered in the parasternal long-axis view,
suprasternal long-axis view, and abdominal aorta. The type of aortic dissection
present is DeBakey type:
A.
B.
C.
D.

I
II
III
IV

462. A patient with a long-standing history of hypertension is sent to the


echocardiography laboratory after developing chest pain that radiated to the back.
The electrocardiogram demonstrated left ventricular hypertrophy, and the chest
roentgenogram revealed a widening of the superior mediastineum. A possible
diagnosis is:
A.
B.
C.
D.

Acute severe aortic insufficiency due to infective endocarditis


Aortic dissection
Mitral valve prolapse
Pericarditis

463. Possible complications of aortic dissection include all the following EXCEPT:
A.
B.
C.
D.

Aortic insufficiency
Left ventricular inflow tract obstruction
Pericardial effusion/tamponade
Progressive enlargement

464. A two-dimensional echocardiographic finding of an aortic intimal flap indicates


aortic:
A.
B.
C.
D.

Aneurysm
Dissection
Insufficiency
Supravalvular stenosis

465. In sinus of Valsalva aneurysm, the coronary cusp most often affected is:
A.
B.
C.
D.

Right coronary cusp


Non coronary cusp
Left coronary cusp
Each coronary cusp is equally affected

466. Associated anomalies of sinus of Valsalva aneurysm include all the following
EXCEPT:
A. Atrial septal defect
B. Bicuspid aortic valve

500 Question Study by Terry Reynolds, BS, RDCS


C. Coarctation of the aorta
D. Ventricular septal defect
467. Types of supravalvular aortic stenosis include:
A.
B.
C.
D.

Discrete fibromuscular stenosis


Hourglass deformity
Hypertrophic obstructive cardiomyopathy
Tunnel aortic valve stenosis

468. The difference between the transmitted and the reflected frequency is known
as the:
A.
B.
C.
D.

Bernoulli equation
Doppler principle
Doppler shift
Gorlin equation

469. Components of the Doppler equation include all the following EXCEPT:
A. The angle between the ultrasound beam and the direction of the blood
flow must be known for accurate measurement of blood flow
B. The transmitted ultrasound frequency is an important determinant of the
Doppler shift detected
C. Propagation speed of sound changes relative to the velocity of red blood
cells
D. The cosine of 0 is 1, and it is assumed in echocardiography that the
recorded velocity has been obtained at a near-parallel intercept angle
470. Minor degrees of tricuspid regurgitation detected by Doppler in structurally
normal hearts:
A.
B.
C.
D.

Are a common finding


Are a rare finding
Depend on respiration
Vary greatly from one echocardiography laboratory to another

471. Pulmonary insufficiency as detected by cardiac Doppler in structurally normal


hearts is:
A.
B.
C.
D.

A rare finding
A common finding
An abnormal finding
Depend on expiration

472. The inability of Doppler ultrasound waves to penetrate prosthetic valves is


called flow:
A.
B.
C.
D.

Masking
Mapping
Convergence
Reverberation

500 Question Study by Terry Reynolds, BS, RDCS


473. As a valve orifice narrows because of stenosis, pressure proximal to the
stenosis will:
A.
B.
C.
D.

Decrease
Equilibrate
Increase with inspiration, decrease with expiration
Increase

474. The pressure drop between two chambers may be calculated by the formula:
A.
B.
C.
D.

CSA TVI
220 pressure half-time
4V2^2
Transmitted frequency received frequency

475. The Doppler formula used to calculate systolic pulmonary artery pressure in a
patient with ventricular septal defect (VSD) is:
A.
B.
C.
D.

BPs-BPd4
BPs-4 (V max VSD^2)
BPd-4 (V max VSD^2)
4 (V1^2)

476. The formula that is used to calculate the peak pressure gradient in coarctation
of the aorta is:
A.
B.
C.
D.

4 (V2^2-V1^2)
4 (V2^2)
220 PHT
CSA TVI

477. The equation that relates the pressure drop across an area of narrowing is the:
A.
B.
C.
D.

Bernoulli equation
Continuity equation
Doppler equation
Velocity ratio equation

478. The simplified Bernoulli equation disregards all the following factors EXCEPT:
A.
B.
C.
D.

Flow acceleration
Proximal velocity
Velocity at the site of stenosis
Viscous friction

479. In patient with aortic valve stenosis, the continuous-wave Doppler recordings
demonstrate a maximum systolic velocity across the aortic valve of 5 m/sec. The
maximum peak instantaneous pressure gradient is:
A. 5 mmHg
B. 25 mmHg

500 Question Study by Terry Reynolds, BS, RDCS


C. 50 mmHg
D. 100 mmHg
480. Right ventricular systolic pressure may be calculated when the following
condition is present:
A.
B.
C.
D.

Aortic insufficiency
MR
Pulmonary insufficiency
TR

481. The formula used to estimate left ventricular end-diastolic pressure (LVEDP)
from the continuous-wave Doppler recording of aortic insufficiency is LVEDP is equal
to:
A.
B.
C.
D.

BPs Vmax AI
BPd Vmax AI
BPd - 4 EDV AI
BPd - 4 EDV AI^2

482. A maximum velocity of 2 m/sec is obtained in a patient with rheumatic mitral


valve stenosis:
A.
B.
C.
D.

2 mmHg
4 mmHg
8 mmHg
16 mmHg

483. The time (in milliseconds) for the pressure difference across a valve to fall to
one-half of the initial peak pressure difference is known as the:
A.
B.
C.
D.

Acceleration half-time
Deceleration half-time
Pressure half-time
Velocity half-time

484. The mitral valve area can be determined by Doppler with the following
formula:
A.
B.
C.
D.

220 pressure half-time


220 deceleration time
Deceleration time pressure half-time
Pressure half-time 220

485. The Doppler hemodynamic parameters that should be evaluated inpatients


with rheumatic mitral stenosis include all the following EXCEPT:

500 Question Study by Terry Reynolds, BS, RDCS


A.
B.
C.
D.

Mitral valve area


Pressure half-time
Pulmonary artery pressure
Regurgitant fraction

486. The formula that allows for calculation of mitral valve area by Doppler is the:
A.
B.
C.
D.

Bernoulli equation
Continuity equation
Gorlin equation
Velocity equation

487. A possible pitfall in the pressure half-time (PHT) method of assessing the
severity of mitral stenosis is concomitant:
A.
B.
C.
D.

Aortic insufficiency
MR
Pulmonary insufficiency
TR

488. Al the following values increase in patients with mitral valve stenosis during
exercise EXCEPT:
A.
B.
C.
D.

Left ventricular end diastolic pressure


Pressure half-time
Systolic pulmonary artery pressure
Transvalvular pressure gradient

489. In patients with significant pure mitral regurgitation, the E velocity of the
mitral valve pulsed-wave Doppler tracing is:
A.
B.
C.
D.

Decreased
Increased with inspiration
Increased
Unaffected

490. The following two-dimensional and cardiac Doppler information is gathered:


the mitral annulus diameter is 4.5 cm, the aortic annulus is 2.0 cm, the mitral time
velocity integral is 20 c, and the aortic time velocity integral is 17 cm. The mitral
regurgitant fraction is:
A.
B.
C.
D.

26.5%
31.8%
53%
83%

500 Question Study by Terry Reynolds, BS, RDCS


491. With aortic valve stenosis and significant aortic insufficiency, the severity of
the aortic valve stenosis by the Doppler pressure gradient may be:
A.
B.
C.
D.

Overestimated
Unaffected
Underestimated
Unpredictable

592. In patients with aortic valve stenosis, the pressure gradients measured by
cardiac Doppler include:
A.
B.
C.
D.

Maximum peak instantaneous gradient and peak-to-peak gradient


Maximum peak instantaneous gradient
Peak-to-mean gradient
peak-to-peak gradient

493. The following data is obtained: left ventricular outflow tract diameter is 2.2
cm, left ventricular outflow tract maximum instantaneous aortic velocity is 6m/sec.
The aortic valve area is:
A.
B.
C.
D.

0.4 cm^2
0.75cm^2
0.68 cm
0.69 cm^2

494. The following data is obtained: left ventricular outflow tract diameter is 2.2
cm, left ventricular outflow tract maximum velocity is 1.1 m/sec, and peak aortic
velocity is 6 m/sec. The aortic velocity ratio is:
A.
B.
C.
D.

0.18
0.18cm
0.18cm^2
0.69cm^2

495. The following data is obtained in a patient with aortic valve stenosis, left
ventricular outflow tract diameter is 2.0 cm, and aortic time velocity integral is 40
cm. The aortic valve area is:
A.
B.
C.
D.

0.3 cm^2
0.75 cm^2
0.9 cm^2
3.14 cm^2

496. The continuous-wave Doppler maximum aortic insufficiency velocity reflects


the:
A. Maximum instantaneous systolic pressure gradient between the aorta and
the left ventricle
B. Maximum peak instantaneous diastolic pressure difference between the
aorta and the left ventricle
C. Mean diastolic pressure gradient between the aorta and the left ventricle

500 Question Study by Terry Reynolds, BS, RDCS


D. Mean systolic pressure gradient between the aorta and the left ventricle
497. The regurgitant fraction (RF%)for aortic insufficiency may be calculated by the
following pulsed-wave Doppler formula:
A.
B.
C.
D.

Aortic RF%= mitral SV-aortic SVmitral SV


Aortic RF%=aortic SV-tricuspid SVaortic SV
Aortic RF%=aortic SV-mitral SVaortic SV
RF% is solely a cardiac catheterization technique that cannot be
duplicated in the echocardiography lab

498. In tricuspid valve stenosis, the Doppler formula used for determining tricuspid
valve area (TVA) is:
A.
B.
C.
D.

Pressure half-time 220


220 pressure half-time
220 deceleration time
0.5 deceleration time

499. Formulas that may be used to calculate the cross-sectional area of an orifice
or vessel through which blood is flowing include all the following EXCEPT:
A.
B.
C.
D.

2r
(D2)
0.785 D
D4

500. The formula used to calculate cardiac output by Doppler is:


A.
B.
C.
D.

EDV ESV
(CSA TVI) HR
(CSA TVI) HR
(CSA TVI) HR BSA

Answers
1. D
2. C
3. A
4. C
5. C
6. D
7. A
8. C
9. D
10.

500 Question Study by Terry Reynolds, BS, RDCS


11.
12.
13.
14.
15.
16.
17.
18.
19.
20.
21.
22.
23.
24.
25.
26.
27.
28.
29.
30.
31.
32.
33.
34.
35.
36.
37.
38.
39.
40.
41.
42.
43.
44.
45.
46.
47.
48.
49.
50.
51.
52.
53.
54.
55.
56.

B
C
D
D
D
C
C
B
A
C
B
B
A
D
B
A
A
A
C
A
A
B
B
D
D
C
D
D
B
C
D
A
A
A
A
A
D
C
A
C
C
A
C
B
B
B

500 Question Study by Terry Reynolds, BS, RDCS


57.
58.
59.
60.
61.
62.
63.
64.
65.
66.
67.
68.
69.
70.
71.
72.
73.
74.
75.
76.
77.
78.
79.
80.
81.
82.
83.
84.
85.
86.
87.
88.
89.
90.
91.
92.
93.
94.
95.
96.
97.
98.
99.
100.
101.
102.

D
A
A
C
C
B
D
C
B
C
B
A
C
C
B
D
B
D
A
B
A
B
A
B
A
B
D
D
C
D
B
B
A
D
D
B
B
D
C
A
B
C
B
C
C
C

500 Question Study by Terry Reynolds, BS, RDCS


103.
104.
105.
106.
107.
108.
109.
110.
111.
112.
113.
114.
115.
116.
117.
118.
119.
120.
121.
122.
123.
124.
125.
126.
127.
128.
129.
130.
131.
132.
133.
134.
135.
136.
137.
138.
139.
140.
141.
142.
143.
144.
145.
146.
147.
148.

C
B
D
D
C
B
C
C
B
B
C
B
C
A
B
A
C
B
C
A
B
D
A
C
B
C
D
C
B
D
B
C
D
B
C
B
C
A
A
A
B
B
C
D
C
B

500 Question Study by Terry Reynolds, BS, RDCS


149.
150.
151.
152.
153.
154.
155.
156.
157.
158.
159.
160.
161.
162.
163.
164.
165.
166.
167.
168.
169.
170.
171.
172.
173.
174.
175.
176.
177.
178.
179.
180.
181.
182.
183.
184.
185.
186.
187.
188.
189.
190.
191.
192.
193.
194.

D
B
D
C
D
C
B
C
B
C
D
D
B
A
A
B
A
A
B
B
B
B
D
D
C
D
C
C
D
D
B
A
B
C
D
D
A
A
A
A
C
B
A
B
D
A

500 Question Study by Terry Reynolds, BS, RDCS


195.
196.
197.
198.
199.
200.
201.
202.
203.
204.
205.
206.
207.
208.
209.
210.
211.
212.
213.
214.
215.
216.
217.
218.
219.
220.
221.
222.
223.
224.
225.
226.
227.
228.
229.
230.
231.
232.
233.
234.
235.
236.
237.
238.
239.
240.

A
A
D
D
B
B
A
C
D
D
B
C
D
A
B
C
D
C
A
D
A
B
B
C
B
D
C
D
D
D
C
B
B
C
C
C
B
C
D
D
C
A
C
C
D
C

500 Question Study by Terry Reynolds, BS, RDCS


241.
242.
243.
244.
245.
246.
247.
248.
249.
250.
251.
252.
253.
254.
255.
256.
257.
258.
259.
260.
261.
262.
263.
264.
265.
266.
267.
268.
269.
270.
271.
272.
273.
274.
275.
276.
277.
278.
279.
280.
281.
282.
283.
284.
285.
286.

C
B
C
D
B
B
B
A
D
A
D
C
D
D
D
C
A
B
D
A
A
B
C
A
A
C
B
A
A
A
C
B
A
C
C
C
D
B
D
B
C
C
B
D
B
B

500 Question Study by Terry Reynolds, BS, RDCS


287.
288.
289.
290.
291.
292.
293.
294.
295.
296.
297.
298.
299.
300.
301.
302.
303.
304.
305.
306.
307.
308.
309.
310.
311.
312.
313.
314.
315.
316.
317.
318.
319.
320.
321.
322.
323.
324.
325.
326.
327.
328.
329.
330.
331.
332.

A
C
D
D
C
D
C
B
C
C
D
D
D
A
D
A
D
D
A
A
D
A
C
D
A
B
B
B
B
A
D
B
C
A
C
A
C
C
D
D
C
A
C
C
D
A

500 Question Study by Terry Reynolds, BS, RDCS


333.
334.
335.
336.
337.
338.
339.
340.
341.
342.
343.
344.
345.
346.
347.
348.
349.
350.
351.
352.
353.
354.
355.
356.
357.
358.
359.
360.
361.
362.
363.
364.
365.
366.
367.
368.
369.
370.
371.
372.
373.
374.
375.
376.
377.
378.

D
A
C
B
C
C
B
A
C
C
D
D
A
A
A
D
C
A
B
B
B
D
A
D
D
C
B
C
B
C
B
D
A
D
A
C
A
D
C
C
B
D
A
B
C
C

500 Question Study by Terry Reynolds, BS, RDCS


379.
380.
381.
382.
383.
384.
385.
386.
387.
388.
389.
390.
391.
392.
393.
394.
395.
396.
397.
398.
399.
400.
401.
402.
403.
404.
405.
406.
407.
408.
409.
410.
411.
412.
413.
414.
415.
416.
417.
418.
419.
420.
421.
422.
423.
424.

D
B
D
A
C
D
C
D
A
C
D
A
A
B
C
D
A
C
A
B
A
B
C
B
D
A
C
A
D
C
B
B
B
D
C
D
B
A
C
A
A
B
C
A
B
C

500 Question Study by Terry Reynolds, BS, RDCS


425.
426.
427.
428.
429.
430.
431.
432.
433.
434.
435.
436.
437.
438.
439.
440.
441.
442.
443.
444.
445.
446.
447.
448.
449.
450.
451.
452.
453.
454.
455.
456.
457.
458.
459.
460.
461.
462.
463.
464.
465.
466.
467.
468.
469.
470.

A
A
A.
A
A
C
A
C
C
B
B
C
D
A
A
B
A
C
A
C
A
B
D
D
C
A
D
A
A
B
B
B
D
D
D
A
A
B
B
B
A
A
B
C
C
A

500 Question Study by Terry Reynolds, BS, RDCS


471.
472.
473.
474.
475.
476.
477.
478.
479.
480.
481.
482.
483.
484.
485.
486.
487.
488.
489.
490.
491.
492.
493.
494.
495.
496.
497.
498.
499.
500.

B
A
D
C
B
A
A
C
D
D
D
D
C
A
D
B
A
B
C
D
A
B
D
A
C
B
C
B
A
B

Das könnte Ihnen auch gefallen